MedSurg II Test 2: Cardiovascular & Peripheral Vascular

अब Quizwiz के साथ अपने होमवर्क और परीक्षाओं को एस करें!

The nurse is caring for a patient diagnosed with mitral valve stenosis. The patient complains of a cough and difficulty breathing when lying down. Upon assessment, the nurse notices the presence of rales and a pulse rate of 110 beats per minute. Which of the following lab results should alert the nurse? ABG - pO2 of 88 mmHg, pCO2 of 40 mmHg ABG - pO2 of 70 mmHg, pCO2 of 65 mmHg Serum Na of 139 mEq/L, HCO3 of 23 mmol/L

*ABG - pO2 of 70 mmHg, pCO2 of 65 mmHg* • ABG - pO2 of 70 mmHg, pCO2 of 65 mmHg. These values indicate hypoxia (low blood oxygen) and hypercapnia (high carbon dioxide). Normal pO2 is 80-100 mmHg and normal pCO2 is 35-45 mmHg. • Mitral valve stenosis can lead to heart failure as evidenced by an increase in pCO2 levels and a decrease in pO2. • Heart failure is a common complication of patients with mitral valve stenosis. This is evident by jugular vein distention, cold clammy skin, tachycardia, orthopnea, increased arterial pCO2, and a decrease in pO2. Rales may be an indication of pulmonary edema that has developed secondary to the congestive heart failure. • Incorrect: The remaining answer choices indicate normal values. Normal sodium range is 135-145 mEq/L. Normal bicarbonate (HCO3) level is 22-30.

The monitor technician informs the nurse that the client has started having premature ventricular contractions every other beat. What should the nurse do first? • Activate the rapid response team. • Assess the client's orientation and vital signs. • Call the health care provider (HCP). • Administer a bolus of lidocaine.

*Assess the client's orientation and vital signs.* The priority action is to assess the client and determine whether the rhythm is life threatening. More information, including vital signs, should be obtained and the nurse should notify the HCP. A bolus of lidocaine may be prescribed to treat this arrhythmia. This is not a code-type situation unless the client has been determined to be in a life-threatening situation.

While monitoring an electrocardiogram (EKG) rhythm strip, the nurse notices that no P-waves can be seen, but the QRS waves are occurring in irregular intervals. The nurse also notes that the heart rate is elevated to 126 beats per minute. This rhythm is recognized as which of the following? Ventricular fibrillation Ventricular tachycardia Atrial fibrillation Atrial tachycardia Submit

*Atrial Fibrillation* •In atrial fibrillation, no P waves can be identified because the sinoatrial (SA) node is not the origin of the electrical impulse. •In atrial fibrillation, multiple areas inside the atria of the heart fire off electrical impulses, preventing the atria from beating effectively, instead only "quivering." •The result is an atrial rate that is sometimes as high as 600bpm, but the beats have a low amplitude and are too fast to see properly on an EKG. •As these multiple electrical impulses travel to the atrioventricular (AV) node, the ventricles are stimulated to contract. These random electrical impulses cause the ventricles to squeeze, causing the ventricular rate to increase to 100-200bpm and the irregular intervals of QRS waveforms on an EKG. •Atrial tachycardia and ventricular tachycardia typically exhibit regular QRS intervals. Ventricular fibrillation does not exhibit regular or identifiable wave forms.

A client is taking spironolactone. Which change in the diet should the nurse teach the client to make when taking this drug? • Restrict sodium intake. • Avoid eating foods high in potassium. • Maintain a fluid intake of 3,000 mL/day. • Incorporate iron-rich foods into the diet.

*Avoid eating foods high in potassium.* Spironolactone is a potassium-sparing diuretic that causes excretion of sodium. When taking this drug, it is important that the client not eat foods high in potassium to avoid elevating serum potassium levels. The client does not need to restrict sodium intake as the drug promotes sodium excretion. Unless contraindicated, the client needs to maintain an adequate fluid intake; however, the client does not need to increase fluid intake to 3,000 mL/day. Spironolactone does not affect iron levels.

A client is recovering from surgical repair of a dissecting aortic aneurysm. Which assessment findings indicate possible bleeding or recurring dissection? • Urine output of 15 ml/hour and 2+ hematuria • Blood pressure of 82/40 mm Hg and heart rate of 125 beats/minute • Urine output of 150 ml/hour and heart rate of 45 beats/minute • Blood pressure of 82/40 mm Hg and heart rate of 45 beats/minute

*Blood pressure of 82/40 mm Hg and heart rate of 125 beats/minute* Assessment findings that indicate possible bleeding or recurring dissection include hypotension with reflex tachycardia (as evidenced by a blood pressure of 82/40 mm Hg and a heart rate of 125 beats/minute), decreased urine output, and unequal or absent peripheral pulses. Hematuria, ↑ urine output, and bradycardia aren't signs of bleeding from aneurysm repair or recurring dissection.

Which assessment findings would the nurse expect to find in the postoperative client experiencing fat embolism syndrome?

*Column B* Fat embolism syndrome is characterized by fever, tachycardia, tachypnea, and hypoxia. Arterial blood gas findings include a partial pressure of oxygen (PaO2) less than 60 mm Hg, with early respiratory alkalosis and later respiratory acidosis.

• Occurs when *cardiac output normal* but body has ↑ metabolic needs • Seen in septicemia • High fever • Anemia • Hyperthyroidism • Rare

*High output failure* (both sided failure)

A patient recently diagnosed with primary hypertension asks the nurse about the risk factors. The nurse lists the risk factors for primary hypertension, including which of the following? *Select all that apply.* Hormonal contraceptives High sodium intake Renal disease Obesity Stress Diabetes mellitus

*High sodium intake* *Obesity* *Stress* • Risk factors for primary hypertension include high salt and fat intake, obesity, stress, alcohol consumption, inactivity, caffeine, and vitamin D deficiency. • Secondary hypertension has identifiable causes, which include but are not limited to renal disease, Cushing's syndrome, hyperthyroidism, hyperaldosteronism, pheochromocytoma, hormonal contraceptives, and diabetes.

The nurse is preparing a patient for a pericardiocentesis for the treatment of cardiac tamponade. How should the patient be positioned? Seated, leaning forward Supine with the head of bed at 45 degrees Supine with Trendelenberg's Right dorsal recumbant

*Supine with the head of bed at 45 degrees* • The patient should be in the supine position, with the head of bed raised to 30-60 degrees. This allows for easier insertion of the needle under the xiphoid process into the pericardial sac.

A patient is experiencing frequent runs of ventricular tachycardia. The nurse should obtain an order from the provider to check serum levels of potassium and magnesium potassium and calcium magnesium and calcium potassium and sodium

*potassium and magnesium* • Electrolyte imbalances that are common causes of ventricular tachycardia are hypomagnesemia (low magnesium), hypokalemia (low potassium), and hyperkalemia (high potassium). • Incorrect: Calcium imbalances may cause QT and ST changes, but not typically ventricular tachycardia. • Incorrect: Sodium imbalances do not cause arrhythmias.

Invasive procedure Kills the problem area

Ablation

Occluded coronary arteries bypassed with pts own veins or synthetic grafts Indicated when no response to medical management *Candidates* Angina with >50% occlusion of L main coronary artery Unstable angina with severe 2 vessel disease Ischemia with heart failure Acute MI with cardiogenic shock Signs of MI after other procedures Valvular disease *Pre-op* Education about the procedure and post op care Demonstration of splinting the chest incision, cough, deep breathe Arm/leg exercises *Post-op* On vent for 3-6 hrs Watch rhythm Check electrolytes - fluid balance Monitor BP, HR, RR LOC Pain Temp Encourage TCDB OOB next day Incision care Ensure someone at home to care for pt/Rehab Limit upper body exercise 6-8 wks Diet - heart healthy

CABG - Coronary artery bypass graft

Reopens the clotted artery Performed within 90 min of MI Sets pts up for CABG Clot retrieval, angioplasty (opening of artery), stent placement Antiplatelet therapy needed Once artery opened and clot resolved should see symptoms resolve CABG (if needed) 5-7 days

CAD Percutaneous Coronary Interventions

The most effective measure the nurse can use to prevent wound infection when changing a client's dressing after coronary artery bypass surgery is to: observe careful handwashing procedures. clean the incisional area with an antiseptic. use prepackaged sterile dressings to cover the incision. place soiled dressings in a waterproof bag before disposing of them.

Correct response: observe careful handwashing procedures. Explanation: Many factors help prevent wound infections, including washing hands carefully, using sterile prepackaged supplies and equipment, cleaning the incisional area well, and disposing of soiled dressings properly. However, most authorities say that the single most effective measure in preventing wound infections is to wash the hands carefully before and after changing dressings. Careful hand washing is also important in reducing other infections often acquired in hospitals, such as urinary tract and respiratory tract infections.

The nurse is assessing a client who has a long history of uncontrolled hypertension. The nurse should assess the client for damage in which area of the eye? iris cornea retina sclera

Correct response: retina Explanation: The retina is especially susceptible to damage in a client with chronic hypertension. The arterioles supplying the retina are damaged. Such damage can lead to vision loss. The iris, cornea, and sclera are not affected by hypertension.

- Occur when another group of cells other than the SA node fire impulses - premature complexes Can be causes by MI, electrolyte imbalances, hypoxia, drug toxicity, hypovolemia, cocaine, illegal drugs, stress, fear, anxiety, caffeine, excessive nicotine and alcohol

Dysrhythmias

• Regular rhythm • Rate normal • QRS duration normal • P wave preceding each QRS PR interval prolonged (>0.20) • All impulses eventually reach ventricles- QRS for every P wave

First Degree Atrioventricular (AV) Block

• When problems started • Smoking • Diet • Family history • Drugs • What symptoms had before (chronic w/ acute flare ups) • Medications- have taken correctly? • Activity level

Heart Failure Patient history - what do we want to know?

An acute systemic vasculitis Self-limited syndrome - causes serious cardiovascular problems in 25% Autoimmune Inflammation by T lymphocytes and neutrophils Coronary dilation or aneurysm may occur

Kawasaki Disease

Fever, chills Headache, malaise, irritability Vomiting, diarrhea, abd pain Joint pain Bilateral conjunctivitis without exudate Dry lips, strawberry tongue Diffuse, erythematous rash Edema of hands, feet Peeling of the perineal region, fingers, and toes Jaundice Hepatomegaly Tachycardia, gallop, or murmur

Kawasaki Disease Assessment

CBC - expect anemia, ↑WBC, ↑platelets C-reactive protein ↑ Echo

Kawasaki Disease Labs/Diagnostics

• none for decades • symptoms happen when *L vent* fails • murmur heard at apex of heart

Mitral Regurgitation *Signs & Symptoms*

The nurse on the previous night shift documented that the lungs of a client with lung cancer were CTA (clear to auscultation) in all fields. While doing the shift assessment the dayshift nurse noticed decreased breath sounds, especially in the right lower lobe. The nurse's best action is to do which of the following? • Report findings to the charge nurse for follow up with the previous shift's nurse regarding the documentation. • Document the findings as the only action as this is expected in clients with lung cancer. • Notify the MD of the change in client status. • Call radiology for an X-ray to confirm findings.

Notify the MD of the change in client status. Pleural effusion is a common complication of lung cancer. Fluid accumulates in the pleural space, impairing transmission of normal breath sounds. Because of the acoustic mismatch, breath sounds are diminished. Documentation of abnormal findings without any follow up is an error in the nursing process. Ordering an X-ray is not an independent nursing action.

*Acute* • Pain relief • Antibiotics • Pericardial drainage • Normally lasts 2-6 wks *Chronic* • Depends on cause --• Cancer - chemo/radiation --• Uremic - dialysis • Definitive Tx --• Removal of the pericardium • Watch for pericardial effusion - fluid - leads to Cardiac tamponade --• Rheumatic carditis - associated with Rheumatic fever - normally from upper respiratory infections - tx antibiotics

Pericarditis *Interventions*

- Endings of the bundle branches - Cells are responsible for rapid conduction of electrical impulses for the ventricle

Purkinje Fibers

0.06-0.10s

QRS

· Atherosclerosis - primary factor · Family Hx · African-American and Hispanic women · Age · ↑ serum lipid levels · Smoking · Sedentary lifestyle · HTN · DM · Obesity · Excessive alcohol/stress

Risk Factors for CAD

• HTN • CAD • Cardiomyopathy • Substance abuse • Valvular disease • Congenital defects • Cardiac infections/inflammations • Dysrhythmias • DM • Smoking • Family history • Obesity • Lung disease • Sleep Apnea • Hyper conditions (ex. Hyperthyroidism)

Risk factors/ Causes of general HF

The nurse is caring for a patient that suffered from a myocardial infarction. While monitoring the patient's electrocardiogram (EKG), the nurse may observe which of the following? *Select all that apply* ST elevation Pathological Q-wave Prolonged PR interval U-wave T-wave inversion Irregular PR interval

ST elevation Pathological Q-wave T-wave inversion

• Regularly irregular normal to slow rate • QRS normal • P wave: QRS = 1:1 then drops QRS complex • PR interval-progressive lengthening then *drop QRS* • Treat: often asymptomatic • PRN: pacemaker, meds to treat bradycardia

Second Degree Type I (Wenckebach) • AKA Mobitz Type I

• Rate normal to slow • P to P interval *consistent* • Constant PR interval • May take > 1 P wave to trigger a QRS • Treat: pacemaker

Second Degree type II (Mobitz type II)

· presence of infection in bloodstream · As it increases- widespread inflammation known as SIRS (Systemic Inflammatory Response Syndrome) --· WBC's produce inflammation markers --· Causes widespread vasodilation and blood pooling

Sepsis

· Usually begins as a bacterial or fungal infection · Infection becomes systemic - especially if pts is immunocompromised from disease process · Progresses up to septic shock and possible death

Sepsis & Septic Shock

Same as adult Watch child carefully May only show symptoms of irritability / not wanting to feed

Sepsis - Peds

· A sepsis induced hypotension despite hydration · Sepsis, SIRS, &MODS (multi-organ dysfunction syndrome) occur · Organ failure is evident · Uncontrolled bleeding occurs - no platelets left · Causes severe hypovolemic shock · Same symptoms of late stage hypovolemic shock · Death rate very high · Prevention

Septic Shock

· Severe sepsis · Sepsis + amplified SIRS · All tissues are involved & hypoxic to some degree · Some organs start to experience cell death & dysfunction · Clots start to form --· This process uses up platelets & causes DIC (disseminated intravascular coagulation) · Stress response triggers the release of glucose from liver causing hyperglycemia

Severe Sepsis

*Cardiogenic*: pump failure *MI* *Hypovolemic*: decreased vascular volume • N/V, hemorrhage, burns *Distributive*: fluid shift • Neural induced - sympathetic nerve impulses decrease - vasodilation -• Loss of perfusion: Head trauma, pain, anesthesia, spinal cord injury • Chemical induced - anaphylaxis, sepsis, capillary leak *Obstructive*: blockage of great vessels • Aortic dissection, pulmonary emboli

Shock *Causes*

Regular rhythm Rate <60 QRS duration normal P wave preceding each QRS PR interval normal - Can be normal or abnormal - look at pt - Symptoms - altered LOC, hypoxic, cool/clammy, decreased BP, dizziness Treat - pacemaker, atropine, fluids, oxygen

Sinus Bradycardia

key: *P wave* = wife *QRS* = husband *Pacer* = counseling *normal sinus rhythm*: the wife (p wave) waits at home for the husband (qrs). the husband (qrs) come homes on time every night. *1st degree av block* the wife (p wave) is waiting at home. the husband (qrs) comes home late every night, but he always comes home and its at the same time every night. *2nd degree block type i (weinkebach)*: the wife (p wave) is waiting at home. the husband (qrs) come home later and later every night until one night he doesn't come at all. *note: husband (qrs) must come home at least 2 nights in a row to see this pattern. *2nd degree av block type ii*: the wife (p wave) is waiting at home. sometimes the husband (qrs) comes home, sometimes he doesn't. when he does come home, its always at the same time. *note: this is usually more serious than type i (weinkebach) and will sometimes require counseling (pacing). *3rd degree av block*: wife (p wave) is no longer waiting at home. she and her husband (qrs) are now both on separate schedules and have no relationship and they are no longer talking. each spouse has a regular, individual schedule. note: this frequently requires counseling in the form of a temporary or permanent pacer.

The Story of the AV Heary Block Family

• Asymmetric ventricular hypertrophy • Genetic • Same symptoms as dilated

*Hypertrophic cardiomyopathy*

Regular rhythm Rate usually slow Absent or *inverted P waves*

AV Nodal - Junctional Rhythm

• Asymptomatic until L vent failure • Bounding pulse • High pitched blowing murmur

Aortic Regurgitation *Signs & Symptoms*

• 4 types - all can lead to sudden death • Dilated cardiomyopathy - most common • Both ventricles dilated • Systolic function impaired

Cardiomyopathy

• CXray • Echo

Heart Failure *Imaging*

BNP ↑

Heart Failure Compensation by the Body *Chemical Responses*

0.08 - 0.10s

P WAVE

0.12-0.20s

P-R interval

• R vent does not completely empty • ↑ pressure & volume in veins Results in peripheral edema

Right Sided Heart Failure

• weight gain • *edema*- liver failure • ascites • Anorexia- liver issues • excessive fluid intake • 3rd- spacing all the fluid • Renal System not working effectively

Right Sided Heart Failure *S/SX*

· WBC's · ↑ D-dimer · *↑ lactic acid* · *↑ prolactin level*

Septic Shock Labs

• A "whole body" response to a lock of oxygen • Inadequate tissue perfusion • Compromised tissue oxygenation

Shock

· ECG, echo, cath, cardiac enzymes · CT scan · H&H · Electolytes · Look for sources of bleed · Cultures · WBC · Coagulation profiles

Shock *Diagnosis*

Regular rhythm Rate 140-220 QRS duration normal P wave often buried PR interval varies depending on site Treat: cardioversion, ablation, oxygen, anti-dysrhythmics (diltiazem, adenosine), vagal Symptoms: palpitations, chest pain, weakness, fatigue, SOB, nervousness, anxiety, hypotension, syncope

Supraventricular Tachycardia (SVT)

• extremely rare • not necessarily lethal • "Twisting" • *LOW MAGNESIUM*

Torsades

Every 3rd Beat PVC Couplet-2 Triplet- 3 >3 = VTach

Trigeminy PVC

A client arrives at the emergency department with severe chest pain and shortness of breath. The client is diaphoretic, pale, and weak. Suddenly, the client collapses and becomes unresponsive. What is the priority action by the nurse? • Initiate chest compressions before ventilations. • Initiate ventilations before chest compressions. • Activate the emergency response team. • Maintain an open airway.

*Initiate chest compressions before ventilations.* This is according to current standards by the American heart and stroke associations (Heart and Stroke Foundation of Canada). The other choices are all in sequence at some point of resuscitation, but current standards are to initiate chest compression, then ventilation.

• Aortic valve leaflets do not close properly --• May be dilated, loose, or deformed • Regurgitation of blood from aorta to L ventricle --• L vent dilates to compensate for amt of blood *Causes* • *Nonrheumatic* conditions --• Congenital, HTN, endocarditis

Aortic Regurgitation

• Most common valve dysfunction • Aortic valve narrows and obstructs L vent outflow • CO cannot increase during exertion • R HF results *Causes* • Congenital, rheumatic disease, atherosclerosis

Aortic Stenosis

Aortic Stenosis Symptoms • Classis symptoms come from fixed cardiac output --• Dyspnea, angina, syncope • Diamond shaped (crescendo-decrescendo) murmur

Aortic Stenosis *Signs & Symptoms*

• no rate Treat: pacemaker, compressions, Epinephrine Do NOT shock

Asystole "flat line"

Ectopic atrial conduction sites Irregular rhythm No distinguishable P wave QRS duration normal Variable rate Treat: Anticoagulants, rate control, cardioversion Afib causes decrease in CO by 20-30% Leads to HF and stroke Risk factors: HTN, Stroke TIA, DM, HR Valve disease, coronary heart disease

Atrial Fibrillation (A-Fib)

Usually regular rhythm variable rate QRS duration normal P wave replaced with F (flutter) waves Rapid atrial depolarization Treat: if symptomatic- slow ventricular response, convert rhythm

Atrial Flutter (A-flutter)

- transmission of impulses - Stimulus moves from cell to cell

Conductivity

Improve O2 Promote nutrition Watch for infection Education Position Fowler's or semi-Fowler's Suction as needed

Congenital Heart Disease Interventions

After evaluating a client for hypertension, a physician orders atenolol, 50 mg P.O. daily. Which therapeutic effect should atenolol have? • Decreased peripheral vascular resistance • Decreased cardiac output and decreased systolic and diastolic blood pressure • Decreased blood pressure with reflex tachycardia • Increased cardiac output and increased systolic and diastolic blood pressure

Decreased cardiac output and decreased systolic and diastolic blood pressure As a long-acting, selective beta1-adrenergic blocker, atenolol decreases cardiac output and systolic and diastolic blood pressure; however, like other beta-adrenergic blockers, it increases peripheral vascular resistance at rest and with exercise. Atenolol may cause bradycardia, not tachycardia.

- Each tiny block is 1mm squared and is equal to 0.04 seconds of time - Five small blocks make up 1 large block - 1 large block = 5mm and 0.20 seconds - 30 large blocks = 6 seconds HR - count the number of QRS complexes in 6 seconds multiply by 10

ECG strip

The nursing priority of care for a client exhibiting signs and symptoms of coronary artery disease should be which of the following? • Educate the client about his symptoms • Decrease anxiety • Enhance myocardial oxygenation • Administer sublingual nitroglycerin

Enhance myocardial oxygenation Enhancing myocardial oxygenation is always the first priority when a client exhibits signs or symptoms of cardiac compromise. Without adequate oxygen, the myocardium suffers damage. A nurse administers sublingualnitroglycerin to treat acute angina pectoris, but its administration is not the first priority. Although educating the client and decreasing anxiety are import in care delivery, neither is a priority when a client is compromised.

• The inability of the heart to work effectively as a pump --• Just doesn't pump anymore • Most common reason for hospitalizations over age of 65 • progressive • Chronic problem w/ acute episodes • Can keep in a stage for a little while

Heart Failure

*Drug management* • ↑ stroke volume • ↓ afterload/preload • ↑ contractility *Diet* • ↓ Na+ • Fluid restriction *Daily weight* • 1kg gain=1L fluid • Same scale / time/ clothing *CPAP* • Respiratory tx for sleep apnea *Pacemaker/Defib* • Class III or IV • ↑ CO, EF *Heart transplant* • Ultimate choice *Physical therapy* • Lifestyle modifications

Heart Failure *Interventions*

Fluid overload *Electrolytes* • Side effect of diuretics • K+, Na+ • Diuretics *BUN/Cr*- Renal function *H/H*- Anemia *BNP*- Fluid retention • Pro BMP ↑ = HF *ABG's* • Oxygen perfusion

Heart Failure *Labs*

• Works to improve CO • Eventually damages the heart Mechanisms

Heart Failure Compensation by the Body

Walls of heart *thicken* to provide more muscle mass for forceful contractions

Heart Failure Compensation by the Body *Myocardial Hypertrophy*

• Microbial infection of endocardium • Seen primarily in --• IV drug use --• Valve replacements --• Systemic infections --• Structural defects • Ports of entry --• Oral cavity --• Skin rashes, lesions, abcesses --• Infections --• Surgery, IV lines • *High mortality rate*

Infective Endocarditis

• Rest • Drug therapy: ABT around 6 wks • Surgery to repair damage only if ABT does not work

Infective Endocarditis *Interventions*

• High fever • Anorexia, wt loss • Murmur (new) • Petechiae • New HF • Arterial embolization (not blood clots but part of vegetation) • Splenic infarct • Positive blood cultures, Echo, TEE to diagnose

Infective Endocarditis *Signs & Symptoms*

ASA, IVIG Fluids Watch for HF Pulses ↓ irritability, calm environment Lip care, popsicles

Kawasaki Disease Inventions

• Mitral valve does not close completely during systole • Backflow of blood • Causes - aging, congenital abnormalities, muscle dysfunction, rheumatic heart disease • Progresses slowly

Mitral Regurgitation

Atria & Ventricles

Pacemaker - Paced Rhythm 2 Dual Pacemaker

Atrial Pacemaker

Pacemaker / Paced Rhythm Atrial Pacemaker

• Pain relieved by sitting up & leaning forward • Friction rub heard on auscultation • Elevated WBC's • Fever • + BC • ST-T spikes or afib *Chronic* - have symptoms of R sided HF

Pericarditis *Signs & Symptoms*

A nurse should monitor a client receiving lidocaine for toxicity. Which signs or symptoms in a client suggest lidocaine toxicity? • Pupillary changes • Confusion and restlessness • Hypertension • Nausea and vomiting

Pupillary changes Confusion and restlessness are signs of lidocaine toxicity. Nausea and vomiting may occur with oral administration of mexiletine or tocainide — other class IB drugs. Pupillary changes and hypertension aren't signs of lidocaine toxicity, although visual changes and hypotension may occur as adverse reactions to class IB drugs.

• *L vent failure* • MI • Pulmonary HTN Too much pressure on pulm. arteries

Right sided Heart Failure *Causes*

indicative of a heart attack

ST Segment Elevation

AKA Mobitz Type I Regularly irregular normal to slow rate QRS normal P wave: QRS = 1:1 then drops QRS complex PR interval- progressive lengthening then drop QRS Treat: often asymptomatic PRN: pacemaker, meds to treat bradycardia

Second Degree Type I (Wenckebach)

• Mild vasoconstriction/ HR increase • Compensatory • Body compensates • moderate Increased HR • Narrowed pulse pressure • RAA activation • Symptoms difficult to detect

Shock Stages *Initial*

• Compensatory mechanisms begin to fail • Vital organ hypoxia • Anoxic/ischemic less vital organs • High mortality if not quickly addressed *Symptoms* • Sense of impending doom, confusion, thirst, rapid/weak pulse, ↓BP, cyanosis, cool/moist skin, anuria, ↓ oxygen saturation

Shock Stages *Progressive*

*Irreversable Stage* • Overwhelming organ damage • multiple organ dysfunction syndrome (MODS) *Symptoms* • LOC, no pulse, cold, slow/shallow respiration, no oxygen saturation

Shock Stages *Refractory*

• Regular rhythm • Slow rate • Prolonged QRS duration • Treatment: pacemaker

Third Degree AV Block (complete HB, complete AV dissociation) Complete dissociation b/w atria and ventricles

• TEE • Echo • cardiac cath • chest x-ray

Valve Diseases (General) *Diagnosis*

*Drug therapy* • Same as heart failure • Anticoags *Valve replacement surgery* • Prosthetic or biologic • Ensure no dental issues *Valvuloplasty* • Balloon dilation *For surgical procedures, implants* • Keep card of implant with you at all times • Avoid activity w/ upper extremities for 3-6 mos • Avoid invasive dental procedures • *No MRI* • Watch for signs of failure (same as HF)

Valve Diseases (General) Treatment

When performing external chest compressions on an adult during cardiopulmonary resuscitation (CPR), the rescuer should depress the sternum: • 0.5 inch (1 cm) • 1 inch (2.5 cm) • 1.5 inches (4 cm) • 2 inches (5 cm)

• 2 inches (5 cm) An adult's sternum must be depressed 2 inches (5 cm) with each compression to ensure adequate heart compression.

The nurse is providing discharge instructions to a patient with congestive heart failure (CHF). Which of the following foods should the nurse tell the patient to avoid? Spinach Ice cream Orange juice Steak sauce

*Steak Sauce* • Patients with CHF should avoid foods high in sodium such as steak sauce. Monitoring/limiting sodium intake is important to help reduce fluid retention. CHF patients should also weigh themselves daily to help monitor fluid retention, which can exacerbate CHF symptoms. • Incorrect: Spinach, ice cream, and orange juice are low in sodium.

Delay history until no symptoms Rapid assessment of symptoms Determine the pain, where coming from, had before, level, intensity, onset, duration, radiation? Chart 38-2 pg 762 BP, HR Associated symptoms ECG - rhythm? Pulses - diminished, skin cool, clammy? RR - lung sounds - crackles/wheezes Heart sounds - S3?

CAD Assessment

Dissolves the clot Alteplase Reteplase Tenecteplase Continuous monitoring Chest pain longer than 30 min/unrelieved by nitro, indications of STEMI on ECG Do not use in NSTEMI Try to start within 30 min of ED arrival Contraindicated in stroke, or recent surgery

CAD Thrombolytic Therapy

Which assessment finding supports the administration of protamine sulfate? • RBCs of 5.4 million/mm3 • aPTT 3.5-5 times normal • Platelets of 152 • INR 8

aPTT 3.5-5 times normal Protamine sulfate is the antidote specific to heparin. The RBC, and platelet levels are normal. Normal aPTT in heparinized clients is 2-2.5 times normal. Therapeutic INR is in the range of 2-3 and relates to therapy with warfarin, not heparin.

The nurse is teaching a client how to apply nitroglycerin topical ointment. Which statement indicates that the client needs additional clarification of the instructions? • "I will carefully massage the ointment into the skin." • "I should remove any remaining old ointment with a tissue before applying a new dose." • "I will use the applicator paper to measure the amount of ointment I should use." • "It is important that I rotate the application sites to avoid skin irritation."

"I will carefully massage the ointment into the skin." The client should not rub or massage the ointment into the skin. The ointment should be allowed to absorb slowly. The client should use the applicator paper to measure the amount of ointment to apply. The client should rotate the application sites to avoid skin irritation. The client should remove any remaining ointment with a tissue before applying a new dose.

A client takes isosorbide dinitrate as an antianginal medication. Which statement indicates that the client understands the adverse effects of the drug? • "I will need to change positions slowly so I will not get dizzy." • "I should take my pulse before taking the medication." • "I should take isosorbide dinitrate with food." • "It is important that I report any swelling in my ankles."

"I will need to change positions slowly so I will not get dizzy." Common adverse effects of isosorbide are light-headedness, dizziness, and orthostatic hypotension. Clients should be instructed to change positions slowly to prevent these adverse effects and to avoid fainting. Ankle swelling is not related to isosorbide administration. The client does not need to take his pulse before taking the medication. The client does not need to take the medication with food.

The nurse is teaching a client about actions to control manifestations of left-sided heart failure. Which statement by the client indicates appropriate understanding? • "I will notify my doctor if I gain 1 pound (0.5 kg) in 24 hours." • "I will use my nitroglycerin tablets if my pulse rises to 90/min." • "If I have trouble breathing, I will sit in my recliner with my head up." • "If I have leg edema I will wear elastic stockings at night."

"If I have trouble breathing, I will sit in my recliner with my head up." The decreased cardiac output that results from left-sided heart failure causes blood to accumulate in the pulmonary system. This produces pulmonary edema and difficulty breathing when lying flat. Weight gain is common with heart failure, but small fluctuations are normal with routine intake and output of food and fluids. Therefore clients are not expected to report a weight change unless it is more than 3 pounds (1.4 kg) in 48 hours. Nitroglycerin tablets are used to treat angina, not a rapid pulse. Elastic stockings may help control peripheral edema, but they do not help left sided heart failure.

A client requested a do-not-resuscitate (DNR) order upon admission to the hospital. He now tells the nurse that he wants the medical team to do everything possible to help him get better and is concerned about the DNR order. Which response by the nurse is best? "Do you want to rescind the DNR, or just change it?" "You know that we will do everything needed to keep you comfortable even though you have the DNR in place." "Have you talked this over with your family?" "It isn't a problem to rescind your DNR order; I'll let your physician know your wishes right away."

"It isn't a problem to rescind your DNR order; I'll let your physician know your wishes right away." Telling the client that it is not a problem to rescind the order is the best response. The client is allowed to rescind a DNR order at any time. The client makes the decision about a DNR order with input from the physician; he does not need to talk to his family. The client needs to have more information regarding the specifics of the nurse's question, but has the right to either rescind or change it at any time.

A patient has been diagnosed with left-sided congestive heart failure, and is confused about return of oxygenated blood from the lungs. To clarify the confusion, the nurse explains all chambers of the heart dealing with blood circulation. The nurse is correct when telling the client Select all that apply. "The blood flows into the left ventricle which pumps it out against high resistance into the systemic circulation." "The left atrium receives oxygenated blood from the lungs." "The blood moves to the left ventricle, which pumps blood into the lungs." "A muscular space called the pericardial space separates the chambers of the right side from the left side." "The right atrium receives deoxygenated blood from the body tissues." "The heart consists of five chambers."

"The blood flows into the left ventricle which pumps it out against high resistance into the systemic circulation." "The left atrium receives oxygenated blood from the lungs." "The right atrium receives deoxygenated blood from the body tissues."

The recipient of a donated organ asks the nurse, "What did the donor die from?" Which response by the nurse is most appropriate? • "I will have the surgeon speak with you." • "Contact between the donor and the recipient is prohibited." • "The transplant coordinator can give you information about the donor's medical history." • "Did you want to send the donor family a thank you card?"

"The transplant coordinator can give you information about the donor's medical history." Confidentiality of the potential donor is always maintained unless the recipient and donor families both sign confidentiality waivers; however, medical history, such as history or hepatitis or HIV infection, is permitted. The transplant coordinator is the liaison for information regarding the donor.

A client comes to the emergency department with symptoms of chest pain radiating down the left arm, dyspnea, and diaphoresis. An electrocardiogram (EKG) shows ST segment elevation and the client is diagnosed with an ST segment-elevation myocardial infarction (STEMI). In order to determine if the client is a candidate for thrombolytic therapy, which question should the nurse ask? • "What time did your chest pain start?" • "Did you take any nitroglycerine before coming to the emergency department?" • "Do you have any allergies?" • "Is this the first time you experienced this type of pain?"

"What time did your chest pain start?" Thrombolytic therapy must be started within 6 hours of the onset of the myocardial infarction (MI). The time the chest pain started is the priority. The nurse can assess for allergies once the time is determined. Nitroglycerine will not impact the administration of thrombolytic therapy.

An anxious client who suffered an acute myocardial infarction is transferred from the coronary care unit to the telemetry unit. The client asks the charge nurse if he can have the same nurse care for him every day. How should the charge nurse respond? • "Different nurses will be assigned to you each day to avoid your becoming dependent on one nurse." • "It's important for you to receive care from a variety of nurses so you can evaluate your care." • "We will try to assign you the same nurse as often as possible." • "It's our policy to rotate client care assignments to ensure quality care for everyone."

* "We will try to assign you the same nurse as often as possible."* The charge nurse should try to accommodate the client's wishes by assigning him a familiar nurse whenever possible. Doing so should help decrease the client's anxiety. Preventing dependency shouldn't be a concern; allaying his anxiety should. The client shouldn't be concerned with evaluating the quality of care rendered by multiple nurses. Providing continuity of care helps ensure quality care.

Which statement would lead the nurse to determine that a client lacks understanding of her acute cardiac illness and the ability to make changes in her lifestyle? • "I already have my airline ticket, so I will not miss my meeting tomorrow." • "These relaxation tapes sound okay; I will see if they help me." • "No more working 10 hours a day for me unless it is an emergency." • "I talked with my husband yesterday about working on a new budget together."

*"I already have my airline ticket, so I will not miss my meeting tomorrow."* Leaving the hospital and immediately flying to a meeting indicate poor judgment by the client and little understanding of what she needs to change regarding her lifestyle. The other statements show that the client understands some of the changes she needs to make to decrease her stress and lead a healthier lifestyle.

A client signed a consent form for participation in a clinical trial for implantable cardioverter-defibrillators. Which statement by the client indicates the need for further teaching before true informed consent can be obtained? • "This implanted defibrillator will protect me against some of those bad rhythms my heart goes into." • "I wonder if there is any other way to prevent these bad rhythms." • "The physician will make a small incision in my chest wall and place the generator there." • "A wire from the generator will be attached to my heart."

*"I wonder if there is any other way to prevent these bad rhythms."* The client wondering if there is another way to prevent the abnormal rhythms indicates that other treatment options weren't discussed with the client. Before participating in a clinical trial, the client must be informed of all other available treatment options. The other statements about implantable cardioverter-defibrillators are all true.

The nurse is providing teaching to a client with newly-diagnosed heart failure. Which description(s) of heart failure should the nurse include? *Select all that apply.* "It is a condition, not a disease." "The heart cannot handle a normal volume of blood." "The heart is unable to pump effectively." "It involves a build up of fluid in the lungs." "It results from too much fluid in the heart." "The heart does not have enough oxygen."

*"It is a condition, not a disease."* *"The heart cannot handle a normal volume of blood."* *"The heart is unable to pump effectively."* • Heart failure is a physiologic state in which the heart cannot pump enough blood to meet the metabolic needs of the body. • The heart fails when, because of intrinsic disease or structural defects, it cannot handle a normal blood volume or, in the absence of disease, cannot tolerate a sudden expansion in blood volume. • Heart failure is characterized by manifestations of volume overload, inadequate tissue perfusion, and poor exercise tolerance.

A client is taking verapamil hydrochloride as an antihypertensive. Which statement made by the nurse instructs the client about an adverse effect of verapamil? "Take your pulse and report any irregular heartbeats." "A low-residue diet will help prevent the occurrence of diarrhea." "You should obtain a complete blood count routinely to monitor for potential bone marrow depression." "Restrict your fluid intake to decrease the chance of developing fluid retention."

*"Take your pulse and report any irregular heartbeats."* Verapamil can cause irregular cardiac rhythms. Clients should be taught to take their pulse and report any irregular heartbeats to their health care provider. Diarrhea is not a problem; constipation is the most common adverse effect of verapamil. Verapamil does not cause bone marrow depression. The client does not need to restrict fluids. Instead, a normal fluid intake is encouraged to prevent constipation.

A client is taking verapamil hydrochloride as an antihypertensive. Which statement made by the nurse instructs the client about an adverse effect of verapamil? • "A low-residue diet will help prevent the occurrence of diarrhea." • "You should obtain a complete blood count routinely to monitor for potential bone marrow depression." • "Take your pulse and report any irregular heartbeats." • "Restrict your fluid intake to decrease the chance of developing fluid retention."

*"Take your pulse and report any irregular heartbeats."* Verapamil can cause irregular cardiac rhythms. Clients should be taught to take their pulse and report any irregular heartbeats to their health care provider. Diarrhea is not a problem; constipation is the most common adverse effect of verapamil. Verapamil does not cause bone marrow depression. The client does not need to restrict fluids. Instead, a normal fluid intake is encouraged to prevent constipation.

While obtaining the patient's vital signs, the nurse notes that the patient's heart rate is 136. Which of the following cardiac effects will likely result from the patient's elevated heart rate? An increase in stroke volume A decrease in ventricular filling time An increase in ventricular filling time A decrease in oxygen demand

*A decrease in ventricular filling time* •Ventricular filling occurs during diastole and is the time when the ventricles fill before pumping the blood to the rest of the body. For this reason, there is reduced end-diastolic filling volume, which decreases stroke volume. Despite the faster heart rate, the decreased stroke volume can impair cardiac output and decrease perfusion, even leading to cardiac failure with extreme heart rates and loss of coordinated atrial contraction. Incorrect options: •Ventricular filling time occurs during diastole and is the time when the ventricles fill with blood before pumping the blood to the rest of the body. Ventricular filling time decreases, not increases, with tachycardia. •Oxygen demand increases, not decreases, as the heart rate increases. •Stroke volume is the amount of blood pumped by the left ventricle with each cardiac contraction

The nurse is caring for a patient who recently underwent an abdominal aortic aneurysm repair. Which complication of this procedure is most common? Acute renal failure Infection Pulmonary embolism Hemorrhage

*Acute Renal Failure* • During a repair of an AAA (abdominal aortic aneurysm), a stent graft is placed in the aorta to support the lumen and reduce pressure on the aneurysm sac. • If the stent graft is placed incorrectly, it could block the renal arteries and cause renal failure. The procedure itself may temporarily reduce blood flow to the renal arteries and cause damage and temporary failure of the kidneys. • Incorrect: Hemorrhage is more common before the AAA repair if the aneurysm ruptures. • Incorrect: Emboli and infection are also possible complications of an AAA repair, but they are less common than renal failure.

The nurse assesses a patient complaining of chest pain that radiates down the left arm. The patient is diaphoretic and short of breath. What should the nurse do next?

*Administer oxygen via nasal cannula, initiate telemetry and check vital signs, and administer nitroglycerin.* • When a patient has symptoms of a myocardial infarction, the nurse should take steps to increase myocardial oxygen supply. This can be done by administered oxygen via nasal cannula and by giving nitroglycerine to vasodilate the coronary arteries, thus increasing perfusion and improving myocardial delivery of oxygen. • Telemetry is the monitoring of electrocardiogram (EKG) and should be evaluated for possible signs of a myocardial infarction, such as ST segment elevation. • Vital signs should be assessed, especially before administering nitroglycerin. • Incorrect: Notifying the physician or cardiac catheterization team before completely assessing the patient is premature. • Incorrect: Norepinephrine (Levophed) is a catecholamine and would increase oxygen demand while decreasing supply.

The nurse is providing interventions to a patient with a deep vein thrombosis (DVT). Which of the following interventions should be avoided? Ambulate frequently. Administer pain medication. Elevate the affected leg. Apply a heat pack to the affected leg.

*Ambulate frequently* • Ambulation is not advised because it is thought that it may increase the risk of the thrombus dislodging and traveling to the lungs as a pulmonary embolism (PE) or leading to a CVA. • Some newer research shows that ambulation does not increase the risk of developing a PE in patients with a DVT, but the current recommendation is still to rest the affected limb and avoid ambulating during recovery. • Incorrect: Heat, pain medication, and elevation of the leg are appropriate nursing interventions when caring for a patient with a DVT.

The nurse is admitting a patient to the telemetry floor. When obtaining the patient history, the nurse notes that the patient has a history of Wolff-Parkinson-White (WPW) syndrome. What does the nurse recognize as the defining characteristic of this conduction syndrome? An increase in the automaticity of the cardiac cells An obstruction of blood flow causing bradycardia An accessory pathway called the bundle of His An accessory pathway called the bundle of Kent

*An accessory pathway called the bundle of Kent* •WPW is characterized by an accessory pathway called the bundle of Kent. •Electrical signals travel down this abnormal accessory conduction pathway (the bundle of Kent) and can stimulate the ventricles to contract prematurely. This causes a type of supraventricular tachycardia referred to as atrioventricular reciprocating tachycardia. Incorrect options: •The bundle of His is the common pathway of atrioventricular (AV) node conduction, not a characteristic of WPW. •WPW is not characterized by an obstruction of blood flow or by bradycardia. •WPW is a conduction syndrome but does not increase the automaticity of the cardiac cells.

A patient is seen at the clinic for a routine physical examination. After the patient is assessed for evidence of peripheral vascular disease (PVD), which of the following tests does the nurse explain is typically used to assist in the diagnosis? Allen's test Cardiac stress test Ankle brachial pressure index Echocardiogram

*Ankle brachial pressure index* • The ankle brachial index (ABI) is the blood pressure ratio between the lower legs and the arms. Blood pressure in the lower legs is normally higher than the arms, and abnormalities indicate narrowing of arteries. Clinical findings that may suggest the presence of PVD include a history of angina with activity, intermittent claudication, and abnormal (weak or absent) pedal pulses. • The formula for ABI is the systolic blood pressure of the ankle (measured at the dorsalis pedis or posterior tibial arteries) divided by the systolic pressure in the arms. It is measured on both sides. • A ratio under 1.0 indicates peripheral vascular disease. • Incorrect: Allen's test is used to assess blood supply to the hand. • Incorrect: Cardiac stress tests are used to measure the heart's ability to respond to stress. • Incorrect: Echocardiogram evaluates the structure and function of the heart muscle, but does not tell if there is peripheral vascular disease present.

A patient with valvular heart disease comes to the emergency department with reports of syncope and dyspnea on exertion. The nurse suspects aortic regurgitation mitral prolapse aortic stenosis mitral stenosis

*Aortic Stenosis* • Aortic stenosis refers to the narrowing of the aortic valve. This causes reduced cardiac output and increased left ventricle pressure. • With exertion, the heart cannot increase cardiac output because of a narrow aortic valve. This can cause syncope due to insufficient perfusion to the brain when it needs the extra oxygen. Syncope is a hallmark sign of aortic stenosis. • Incorrect: Aortic regurgitation, mitral stenosis, and mitral regurgitation do not cause syncope.

A patient with aortic regurgitation is seen in the clinic for a regular checkup. The nurse assesses the patient and hears an Austin Flint murmur, which is heard best in which location? Apex of the heart Second intercostal space, left sternal border Second intercostal space, right sternal border Carotid arteries

*Apex of the heart* • An Austin Flint murmur is best heard over the apex of the heart. It is described as low-pitched and rumbling and can be heard during mid- to late diastole. This murmur is associated with severe aortic regurgitation. • Incorrect: The second intercostal space on the left sternal border is the best location for hearing pulmonic stenosis. • Incorrect: The second intercostal space on the right sternal border is the best location for hearing aortic stenosis • Incorrect: Carotid arteries can produce a bruit if occluded, which may sound like a murmur. Aortic stenosis murmurs may also at times radiate to the carotid arteries.

The nurse is concerned about the risks of hypoxemia and metabolic acidosis in a client who is in shock. What finding should the analyze for evidence of hypoxemia and metabolic acidosis in a client with shock? • Oxygen saturation level • Arterial blood gas (ABG) findings • Red blood cells (RBCs) and hemoglobin count findings • White blood cell differential

*Arterial blood gas (ABG) findings* Analysis of ABG findings is essential for evidence of hypoxemia and metabolic acidosis. Low RBCs and hemoglobin correlate with hypovolemic shock and can lead to poor oxygenation. An elevated white blood cell count supports septic shock. Oxygen saturation levels are usually affected by hypoxemia but cannot be used to diagnose acid-base imbalances such as metabolic acidosis.

A patient has just returned to the nursing unit after undergoing a cardiac catheterization. Which of the following nursing interventions should the nurse perform first? Administer pain medication. Check the insertion site for urticaria, as this could be an early sign of anaphylaxis. Administer fluids to flush out the contrast dye. Assess insertion site for bleeding or hematoma formation.

*Assess insertion site for bleeding or hematoma formation.* • The insertion site, usually the femoral artery, should be checked frequently for signs of bleeding or hematoma formation. The insertion site is at risk for bleeding due to the high pressure of the femoral artery and the large diameter of the catheter. • Fluids should be encouraged to flush out the dye used during the procedure, but the first priority is to assess the insertion site for bleeding. • Urticaria and anaphylaxis are not common after cardiac catheterization. • Pain medication should be administered as prescribed, but assessing for bleeding takes priority. Administer pain medication.

nurse on the cardiac telemetry floor is monitoring the electrocardiogram (EKG) of a patient admitted with an elevated heart rate. The nurse notes that the EKG shows an atrial rate of 300 with the P waves in a "saw-tooth" pattern. The AV conduction ratio is 3:1 with a ventricular response rate of 100. The nurse knows this rhythm is which of the following? Supraventricular tachycardia Paroxysmal atrial tachycardia Atrial flutter Atrial fibrillation

*Atrial Flutter* •Atrial flutter occurs due to a circular pathway in the right atrium where the sinoatrial (SA) node is located. •The electrical impulse conducts through the cells that are polarized and ready to receive an impulse, resulting in continual re-firing. The AV node will not conduct each of these impulses to the ventricles, so ventricular depolarization stays constant. The result is the appearance of "flutter" P waves with normal QRS waves. •Conduction ratio is calculated based on the number of flutter waves for each QRS wave.

While reviewing an electrocardiogram (EKG), the nurse knows that the "P" wave represents which of the following? Ventricular repolarization Ventricular depolarization Atrial repolarization Atrial depolarization

*Atrial depolarization* •Atrial depolarization: P waves occur during atrial "squeeze" or contraction. •The prefix "de" means "away" so as the atria squeeze, blood is moved "away" and into the ventricles. •P waves are smaller as compared to the QRS waves because the atria are smaller heart chambers than are the ventricles.

A patient who underwent an axillofemoral bypass graft is receiving instructions from the nurse. The nurse should include which of the following information? • Avoid exercise. • Avoid belts. • Avoid suspenders. • Avoid standing for prolonged periods.

*Avoid belts.* • An axillofemoral bypass is a surgical revascularization procedure used to treat symptomatic aortoiliac occlusive disease for patients who cannot undergo aortofemoral intervention. The axillary artery is connected to the femoral artery via an artificial graft in order to restore perfusion to the lower extremity. • The axillary artery is connected via a long, flexible graft to the femoral artery below (and another graft may shunt blood to the opposite femoral artery) in order to restore perfusion to the lower extremities. • Wearing a waist belt would put pressure across the abdomen, possibly occluding the bypass graft and cutting off circulation to the lower extremities. • Suspenders are preferred over a belt. • Exercise and standing do not need to be avoided.

A client, diagnosed with insulin-dependent diabetes and acute heart failure, was admitted to the hospital for medical management. What are the appropriate pharmacological interventions to help reduce myocardial workload and improve ventricular pump performance? *Select all that apply.* Beta-blockers Dobutamine IV Morphine IV Leukotriene modifiers Dantrolene

*Beta-blockers* *Dobutamine IV* *Morphine IV* • Morphine IV is often used for clients with acute heart failure. It is not only an anxiolytic and analgesic, but its most important effect is venodilation, which reduces preload. • Beta-blockers are used to inhibit the effects of the sympathetic nervous system and reduce the oxygen needs of the myocardium. • Dobutamine is a very useful medication for heart failure because it produces strong beta-stimulatory effects within the myocardium. It increases heart rate, atrioventricular conduction, and myocardial contractility. • Leukotriene modifiers are used in the treatment of both acute and chronic asthma. This also does not help with workload or pump performance. • Dantrolene is classified as a direct-acting skeletal muscle relaxant. It is currently the only specific and effective treatment for malignant hyperthermia.

A client is returning from the operating room after inguinal hernia repair. The nurse notes that he has fluid volume excess from the operation and is at risk for left-sided heart failure. Which sign or symptom indicates left-sided heart failure? • Bibasilar crackles • Right upper quadrant pain • Dependent edema • Jugular vein distention

*Bibasilar crackles* Bibasilar crackles are a sign of alveolar fluid, a sequelae of left ventricular fluid, or pressure overload and indicate left-sided heart failure. Jugular vein distention, right upper quadrant pain (hepatomegaly), and dependent edema are caused by right-sided heart failure, usually a chronic condition.

A client is receiving nitroglycerin ointment to treat angina pectoris. The nurse evaluates the therapeutic effectiveness of this drug by assessing the client's response and checking for adverse effects. Which vital sign is most likely to reflect an adverse effect of nitroglycerin? • Pulse rate of 84 beats/minute • Respiration 26 breaths/minute • Blood pressure 84/52 mm Hg • Temperature of 100.2° F (37.9° C)

*Blood pressure 84/52 mm Hg* Hypotension and headache are the most common adverse effects of nitroglycerin. Therefore, blood pressure is the vital sign most likely to reflect an adverse effect of this drug. The nurse should check the client's blood pressure 1 hour after administering nitroglycerin ointment. A blood pressure decrease of 10 mm Hg is within the therapeutic range. If blood pressure falls more than 20 mm Hg below baseline, the nurse should remove the ointment and report the finding to the physician immediately. An above-normal heart rate (tachycardia) is a less common adverse effect of nitroglycerin. Respiratory rate and temperature don't change significantly after nitroglycerin administration.

The nurse instructs a client with coronary artery disease in the proper use of nitroglycerin. The client has had 2 previous episodes of coronary artery disease. At the onset of chest pain, what should the client do? • Call 911 when three nitroglycerin tablets taken every 5 minutes are not effective. • Call 911 when five nitroglycerin tablets taken every 5 minutes are not effective. • Take one tablet and then immediately call 911. • Go to the emergency department if two nitroglycerin tablets taken 5 minutes apart are not effective.

*Call 911 when three nitroglycerin tablets taken every 5 minutes are not effective.* Nitroglycerin tablets should be taken 5 minutes apart for three doses; if this is ineffective, 911 should be called to obtain an ambulance to take the client to the emergency department. The client should not drive or have a family member drive the client to the hospital.

A child with heart disease starts on oral digoxin. When preparing to administer the medication, what should the nurse do first? • Check the last serum electrolyte results for the child. • Verify the dosage with a licensed practical/vocational nurse (LPN/VN) who is working that day. • Ask the mother if she is willing to administer the medication. • Teach the mother how to measure the child's heart rate.

*Check the last serum electrolyte results for the child.* It is most important to know the child's serum potassium level when administering digoxin. Digoxin ↑ contractility of the heart and ↑ renal perfusion, resulting in a diuretic effect with ↑ loss of potassium and sodium. Hypokalemia ↑ the risk of digoxin toxicity. Verifying the dosage is specified by facility policy and varies among facilities. Although the child may take the medication better from the mother than from the nurse, asking the mother to give the medication is not necessary. In addition, this would be done after the nurse has checked the electrolyte levels. Teaching the parent how to measure the child's heart rate can be done at any time, not necessarily when preparing to give digoxin.

The nurse instructs a patient on the symptoms of hypertension. Additional education is required if the patient states that which of the following is a symptom of high blood pressure? Chest pain Feeling of fullness in the head Headache Blurred vision

*Chest Pain* • Incorrect: Hypertension is rarely accompanied by symptoms. When symptoms do occur, they include headache, tinnitus, blurred vision, dizziness, and a feeling of fullness in the head. Correct: Chest pain is not a symptom of hypertension, but can be an emergent symptoms indicating hypertensive crisis or myocardial infarction.

*A* - patients at ↑ risk for developing heart failure *B* - patients w/ cardiac structural abnormalities or remodeling who have not developed HF symptoms *C*- Patients w/ current or prior symptoms of heart failure (symptoms appear) *D* - Patients w/ refractory end-stage heart failure

*Classification of HF* - From American Heart Association & American College of Cardiology

The nurse is assessing a client with an atrial septal defect (ASD). Which requires immediate nursing intervention? • Fixed split S2, which does not vary with respiration • Client not taking his angiotensin-converting enzyme inhibitor this morning • Client having an uneven smile and facial droop • Client having tachycardia at a rate of 100 beats/min

*Client having an uneven smile and facial droop* A fixed S2 split is the hallmark of ASD. The neurologic finding of a facial droop could indicate embolization and stroke; the nurse should notify the healthcare provider immediately. If the client has missed a medication, the nurse should measure the vital signs and administer the medication as soon as possible; however, symptoms of stroke are the priority. The nurse should further assess tachycardia to determine the underlying cause, such as pain or fever, before intervening.

A client with septic shock has continued to deteriorate and has become unresponsive. The nurse has inserted an intravenous line and an oral airway. Which of the following is the highest priority for the nurse at this time? • Check the surgical dressing to ensure that it is intact. • Confirm the placement of the oral airway. • Examine the IV site for infiltration. • Monitor temperature every 4 hours.

*Confirm the placement of the oral airway.* Confirming the placement of the oral airway ensures a patent air passage. Oxygen is essential for life, so this action takes priority. Other answers do not reflect ABC priority based on client unresponsiveness.

A nurse assessing a client who underwent cardiac catheterization finds the client lying flat on the bed. His temperature is 99.8° F (37.7° C). His blood pressure is 104/68 mm Hg. His pulse rate is 76 beats/minute. The nurse assesses the limb and detects weak pulses in the leg distal to the puncture site. Skin on the leg is cool to the touch. The puncture site is dry, but swollen. What is the most appropriate action for the nurse to take? • Document the findings and recheck the client in 1 hour. • Slow the I.V. fluid to prevent any more swelling at the puncture site. • Contact the physician and report the findings. • Encourage the client to perform isometric leg exercise to improve circulation in his legs.

*Contact the physician and report the findings.* The client is probably developing a hematoma at the puncture site. The decreased pulses, swelling, and cool temperature in the leg are all classic signs that blood flow to that extremity is compromised. The nurse should notify the physician immediately to preserve the blood flow in the client's leg. Documenting findings and checking the client again in 1 hour, slowing the I.V. fluid, and encouraging the client to perform isometric leg exercises aren't appropriate actions for the nurse to take at this time.

A nurse is caring for a client with type 2 diabetes who has had a myocardial infarction (MI) and is reporting nausea, vomiting, dyspnea, and substernal chest pain. Which of the following is the priority intervention? • Reduce the nausea and vomiting and stabilize the blood glucose. • Control the pain and support breathing and oxygenation. • Decrease the anxiety and reduce the workload on the heart. • Monitor and manage potential complications.

*Control the pain and support breathing and oxygenation.* Support of breathing and ensuring adequate oxygenation are the two most important priorities. Reducing the substernal pain is also important because upset and anxiety will increase the demand for oxygen in the body. Controlling nausea, vomiting, and anxiety are all secondary in importance. Prevention of complications is important following initial stabilization and control of pain.

A client receiving a continuous infusion of lidocaine for ventricular dysrhythmias states "I am so tired. Even my vision is blurry." What is the nurse's best action? • Administer zolpidem. • Decrease the lidocaine infusion rate. • Cluster activities to allow the client uninterrupted rest time. • Ask the client when their last eye exam was completed.

*Decrease the lidocaine infusion rate.* Side effects of lidocaine include lightheaded, euphoria, shaking, low blood pressure, drowsiness, confusion, weakness, blurry or double vision, and dizziness. Serious reactions such as seizures, bradycardia, and heart block are possible if lidocaine reaches toxic levels. The nurse should recognize these potential adverse effects and the lidocaine infusion should be decreased while lidocaine blood levels are checked to determine if the cause of the tiredness and blurred vision is a lidocaine toxicity. Knowing when the client's last eye examination was completed and allowing the client to rest or administering zolpidem will not address the problem of potential lidocaine toxicity and may lead to the more serious toxic reactions.

A client is recovering from coronary artery bypass graft (CABG) surgery and begins to experience chest pain, shortness of breath, and tachycardia. Further assessment reveals a widened QRS complex and an elevated ST segment. Which nursing diagnosis takes highest priority at this time? • Decreased cardiac output related to depressed myocardial function • Anxiety related to an actual threat to health status and pain • Activity intolerance related to imbalance between oxygen supply and demand • Acute Pain related to impaired electrical conduction

*Decreased cardiac output related to depressed myocardial function* For a client recovering from CABG surgery and experiencing these symptoms, decreased cardiac output is the most important nursing diagnosis. Complications of CABG include hemorrhage, dysrhythmias, and myocardial infarction (MI) leading to decreased cardiac output. Anxiety, activity intolerance, and acute pain may be relevant, but take lower priority at this time; maintaining cardiac output is essential to sustaining the client's life.

The nurse is providing discharge teaching to a patient who underwent a coronary artery bypass graft (CABG). Which of the following should the nurse explain is common in the weeks following surgery, but will likely resolve without treatment? Edema Nausea Depression Dizziness

*Depression* • Depression is very common after undergoing a CABG. It usually resolves within several weeks after recovery and generally does not require medical intervention. • Incorrect: Dizziness may signal a decrease in cardiac output. This is an abnormal finding and requires immediate attention. • Incorrect: Nausea is not an expected side effect except in weeks following surgery. • Incorrect: Edema after a CABG may indicate heart failure and should not be ignored.

A client comes to the emergency department complaining of chest pain. An electrocardiogram (ECG) reveals myocardial ischemia and an anterior-wall myocardial infarction (MI). Which ECG characteristic does the nurse expect to see? • Prolonged PR interval • Absent Q wave • Elevated ST segment • Widened QRS complex

*Elevated ST segment* Ischemic myocardial tissue changes cause elevation of the ST segment, an inverted T wave, and a pathological Q wave. A prolonged PR interval occurs with first-degree heart block, the least dangerous atrioventricular heart block; this disorder may arise in healthy people but sometimes results from drug toxicity, electrolyte or metabolic disturbances, rheumatic fever, or chronic degenerative disease of the conduction system. An absent Q wave is normal; an MI may cause a significant Q wave. A widened QRS complex indicates a conduction delay in the His-Purkinje system.

The nurse is assessing a patient for coronary artery disease (CAD) risk factors. Which of the following modifiable risk factors has the most impact on the development of CAD? Type 1 diabetes mellitus Elevated high density lipoproteins (HDLs) Elevated low density lipoproteins (LDLs) Elevated triglycerides

*Elevated low density lipoproteins (LDLs)* • Elevated LDLs are the biggest modifiable risk factor in the development of CAD. LDLs transport cholesterol to the artery walls, where they develop into atherosclerotic plaques. • Incorrect: Type 1 diabetes is a serious but non-modifiable risk factor. • Incorrect: Elevated HDLs are protective against CAD. • Incorrect: Elevated triglyceride levels are linked to the development of CAD, but they are not as important as LDLs. The risk is partially accounted for by a strong inverse relationship between triglyceride levels and HDL levels.

The nurse is caring for a patient with an implanted pacemaker. While observing the cardiac monitor, the nurse notes extra pacer spikes that are not followed by a heartbeat. The nurse suspects which condition? Failure to pace Asystole Failure to capture Failure to sense

*Failure to capture* • Failure to capture is when the pacer fires a signal, but the heart does not respond with a beat. This is often caused by pacer lead migration on the epicardium. • Incorrect: Failure to pace is when the pacer fails to fire a signal when it should. This often leads to bradycardia and hypotension because the heart cannot beat at the same rate as the pacer. • Incorrect: Failure to sense is when the pacer cannot sense the heartbeat, leading to poorly-timed firing by the pacer. • Incorrect: Asystole is the absence of a heart rate.

As part of a comprehensive cardiovascular examination, the nurse palpates the apical pulse. At what location should the nurse palpate? Fifth intercostal space, left midclavicular line Third intercostal space, left midclavicular line Fifth intercostal space, left sternal border Fourth intercostal space, left sternal border

*Fifth intercostal space, left midclavicular line* • The apical pulse is palpated at the fifth intercostal space on the left midclavicular line. This is also known as the point of maximal impulse (PMI).

A white male, age 43, with a tentative diagnosis of infective endocarditis is admitted to an acute care facility. His medical history reveals diabetes mellitus, hypertension, and pernicious anemia; he underwent an appendectomy 20 years earlier and an aortic valve replacement 2 years before this admission. Which history finding is a major risk factor for infective endocarditis? • Race • Age • History of diabetes mellitus • History of aortic valve replacement

*History of aortic valve replacement* A heart valve prosthesis such as an aortic valve replacement is a major risk factor for infective endocarditis. Other risk factors include a history of heart disease (especially mitral valve prolapse), chronic debilitating disease, I.V. drug abuse, and immunosuppression. Although race, age, and a history of diabetes mellitus may predispose a person to cardiovascular disease, they aren't major risk factors for infective endocarditis.

The nurse is caring for a patient in the early stages of heart failure. The family is curious as to how the body adapts to heart failure. The nurse knows that during the early stages of heart failure, which specific compensatory mechanisms occur? Hypotension stimulates the baroreceptors to decrease sympathetic activity. Hypotension stimulates baroreceptors to increase sympathetic activity. Impaired renal perfusion inhibits aldosterone release. Decreased cardiac output inhibits the release of antidiuretic hormone (ADH) by the pituitary gland.

*Hypotension stimulates baroreceptors to increase sympathetic activity.* • As arterial blood pressure falls, baroreceptors of the carotid and aorta are stimulated. This causes a sympathetic release of catecholamines, resulting in vasoconstriction and an increased heart rate to compensate. • The compensatory mechanism increases peripheral vascular resistance and also the workload of the heart. This can worsen heart failure if not treated. • Incorrect: Decreased cardiac output will increase ADH release, leading to fluid retention. • Incorrect: Impaired renal perfusion will stimulate aldosterone release, leading to additional fluid and sodium retention.

The nurse is assessing a patient scheduled for surgery later today. The patient is complaining of chest and shoulder pain, perspiration, and shortness of breath. Which of the following should be done? Sign the pre-operative checklist and call for transport to the operating room. Check the patient's electrolytes and perform a complete blood count (CBC). Initiate protocol for the medical emergency team (MET) and notify the physician. Request a chest X-ray.

*Initiate protocol for the medical emergency team (MET) and notify the physician.* • These symptoms are consistent with an acute myocardial infarction (MI), which is a medical emergency. The hospital or facility should have a protocol in place to assist the nurse for a patient experiencing a suspected MI. The MET team usually has standing orders for blood work, electrocardiogram (EKG), oxygen, nitrogen and/or morphine administration. This workup can begin while the provider is being contacted. • Using the A-B-C Framework, circulation and thus oxygenation to the heart is impaired, but none of the options are for interventions that address that. The initial assessment has already indicated signs of an MI. Even though it is not stated, oxygen should be administered and the MET team called or other hospital protocol followed while the physician is being notified. • Incorrect: Labs, X-rays, and other tests need to be ordered by the physician (or done with standing orders by the MET team staff). The nurse should anticipate that an EKG would be the first test obtained. A chest X-ray does not diagnose an acute MI, but an EKG would. • Incorrect: The patient should not be sent to the operating room while he is experiencing a suspected MI. The provider will re-evaluate whether or not to continue with surgery after immediate concerns of an MI are identified and treated.

A patient is admitted to the hospital due to a myocardial infarction. Which of the following interventions or devices would increase coronary perfusion and decrease cardiac workload? Intra-aortic balloon pump Pacemaker Dobutamine Therapeutic hypothermia

*Intra-aortic balloon pump* • An intra-aortic balloon pump inflates during diastole to increase perfusion to the coronary arteries and organs and deflates during systole to decrease afterload. • The timed inflation and deflation contribute to increased myocardial oxygen supply while decreasing myocardial oxygen demand. • Incorrect: Pacemakers do not increase coronary perfusion or decrease workload. • Incorrect: Dobutamine will increase cardiac workload by increasing contractility and cardiac output. • Incorrect: Therapeutic hypothermia will decrease oxygen demands, but will not improve coronary perfusion.

A patient is admitted to the hospital with a myocardial infarction (MI). When assessing the patient for pain, the nurse is most likely to observe referred pain in what location? Left lower quadrant of the abdomen Sternum Head Left shoulder

*Left shoulder* • Referred pain is pain that is felt in an area other than the stimulus. • During a myocardial infarction, pain is commonly felt in the chest, but pain also refers (or radiates) to the left shoulder or arm, neck, or jaw. • Women usually experience the same chest pain as men, but are more likely than men to have pain in the back and jaw, fatigue, dizziness or lightheadedness, and shortness of breath with nausea or vomiting. Women can also have no chest pain but feel pressure in the upper back or upper abdomen. • Women are more likely than men to die from a heart attack. • Sternal (chest) pain would not be considered referred pain, but rather primary pain during an acute MI. • Pain in the head or left lower quadrant of the abdomen are not common locations for referred pain associated with a MI.

A nurse is teaching a client who receives nitrates for the relief of chest pain. Which instruction should the nurse emphasize? • Repeat the dose of sublingual nitroglycerin every 15 minutes for three doses. • Store the drug in a cool, well-lit place. • Lie down or sit in a chair for 5 to 10 minutes after taking the drug. • Restrict alcohol intake to two drinks per day.

*Lie down or sit in a chair for 5 to 10 minutes after taking the drug.* Nitrates act primarily to relax coronary smooth muscle and produce vasodilation. They can cause hypotension, which makes the client dizzy and weak. The nurse should instruct the client to lie down or sit in a chair for 5 to 10 minutes after taking the drug. Nitrates are taken at the first sign of chest pain and before activities that might induce chest pain. Sublingual nitroglycerin is taken every 5 minutes for three doses. If the pain persists, the client should seek medical assistance immediately. Nitrates must be stored in a dark place in a closed container because sunlight causes the medication to lose its effectiveness. Alcohol is prohibited because nitrates may enhance the effects of the alcohol.

A nurse in the critical care unit is monitoring the patient's electrocardiogram (EKG). The nurse notes that the PR interval is growing increasingly prolonged before each QRS. Then the nurse notes that a P-wave occurs with no QRS present. The nurse recognizes this rhythm as which one of the following? Mobitz type I second-degree AV block Mobitz type II second-degree AV block First-degree AV block Third-degree atrioventricular (AV) block

*Mobitz type I second-degree AV block* •In Mobitz type I second-degree AV block, the block occurs in the AV node itself. Ischemia (decreased oxygen) to the inferior heart wall is the usual cause. •Mobitz type I second-degree block is often transient, meaning that it occurs and resolves intermittently. The notable feature of this heart block is a PR interval that continues to prolong until no QRS is generated. •Mobitz type I second-degree AV block is also called a Wenckebach block. A good way to remember this is the rhyme: "Longer, longer, longer, drop, now you have a Wenckebach."

Upon assessment of third-degree heart block on the monitor, what should the nurse do first? • Call a code. • Begin cardiopulmonary resuscitation. • Place transcutaneous pads on the client. • Prepare for defibrillation.

*Place transcutaneous pads on the client.* Transcutaneous pads should be placed on the client with third-degree heart block. For a client who is symptomatic, transcutaneous pacing is the treatment of choice. The hemodynamic stability and pulse should be assessed prior to calling a code or initiating CPR. Defibrillation is performed for ventricular fibrillation or ventricular tachycardia with no pulse.

While reviewing the patient's chart, the nurse notes that the patient takes digoxin for congestive heart failure (CHF). Which of the following early findings on the patient's electrocardiogram (EKG) would alert the nurse to a possible digoxin toxicity? Bradycardia Premature ventricular contractions (PVCs) Tachycardia Premature atrial contractions (PACs)

*Premature ventricular contractions (PVCs)* •Premature ventricular contractions are the most common EKG abnormality seen in early digoxin toxicity. Digoxin toxicity causes a malfunction in the sodium/potassium/ATPase pump of the cardiac cell, causing PVCs. Incorrect options: •PACs arise from the atria and cause the sinoatrial (SA) node (the natural pacemaker of the heart) to fire prematurely. While PACs may eventually be seen in a digoxin toxicity, this dysrhythmia is not the most frequently seen early dysrhythmia of digoxin toxicity. •Tachycardia, or a heart rate greater than 100 beats per minute, is a common cardiac dysrhythmia but is not typically seen in early digoxin toxicity. •Bradycardia, or a heart rate less than 60 beats per minute, is often a late symptom of digoxin toxicity, not an early symptom.

A postoperative client is receiving heparin after developing thrombophlebitis. The nurse monitors the client carefully for bleeding and other adverse effects of heparin. If the client starts to exhibit signs of excessive bleeding, the nurse should expect to administer an antidote that is specific to heparin. Which agent fits this description? • Phytonadione (vitamin K) • Protamine sulfate • Thrombin • Plasma protein fraction

*Protamine sulfate* Protamine sulfate is the antidote specific to heparin. Phytonadione (vitamin K) is the antidote specific to oral anticoagulants such as warfarin. (Heparin isn't given orally.) Thrombin is a hemostatic agent used to control local bleeding. Plasma protein fraction, a blood derivative, supplies colloids to the blood and expands plasma volume; it's used to treat clients who are in shock.

A nurse on the cardiac step-down unit is caring for a patient who is recovering from a myocardial infarction and drug eluding stent placement. The nurse is preparing to educate the patient on the Dietary Approaches to Stop Hypertension (DASH) diet which includes which of the following recommendations? • Increase physical activity daily so that sodium intake can remain unchanged • Reduce sodium intake to less than 2300 mg daily, which can help lower blood pressure • Decrease potassium and magnesium intake to prevent possible cardiac arrhythmias • Include at least 2300 mg of sodium intake daily to maintain adequate blood fluid volume

*Reduce sodium intake to less than 2300 mg daily, which can help lower blood pressure* •The DASH diet is the "Dietary Approaches to Stop Hypertension" diet. •An important part of the DASH diet approach is a low sodium diet with recommendations of intake less than 2300 mg of sodium daily. •People with other co-morbidities like diabetes & heart disease are recommended to have less sodium intake. Water is drawn to sodium (think about how thirsty you get when eating salty snacks like potato chips), so increased sodium intake causes fluid retention. This increased fluid retention can cause increased blood pressure because more fluid is circulating in the bloodstream and blood vessels. Because of this correlation, decreasing sodium intake can help reduce the amount of excess fluid in the blood, and blood pressure will also decrease.

• Very rare • Stiff ventricles • Same symptoms as HF • Poor prognosis

*Restrictive cardiomyopathy*

A client with deep vein thrombosis has been receiving warfarin for 2 months. The client is to go to an anticoagulant monitoring laboratory every 3 weeks. The last visit to the laboratory was 2 weeks ago. The client reports bleeding gums, ↑ bruising, and dark stools. What should the nurse should instruct the client to do? • Decrease the dose of the warfarin. • Return to laboratory for analysis of prothrombin times. • Decrease the amount of vitamin K in the diet. • Notify the health care provider (HCP) about the bleeding.

*Return to laboratory for analysis of prothrombin times.* These symptoms suggest that the client is receiving too much warfarin; the client should return to the laboratory and have a blood sample drawn to determine the prothrombin levels and have the dosage of warfarin adjusted. The diet can influence clotting, but the client needs to first have the prothrombin levels checked. It is not necessary to contact the HCP; the client should return to the laboratory first, and the results of the prothrombin time will be reported to the HCP.

The nurse is assessing a patient with coronary artery disease who has been hospitalized for 3 days. The nurse notes 3+ edema of the bilateral lower extremities and crackles in the lungs. Which of the following should the nurse do next? Review the intake and output records since admission. Request an increase in diuretic dosage. Request a sodium-restricted diet. Request an order for daily weights.

*Review the intake and output records since admission.* • Remember, unless there is a need for immediate intervention, then conduct an assessment first. • The nurse should review the patient's intake and output records to determine if there is a disparity. Input greater than output will cause fluid overload, as evidence by edema and crackles. • Incorrect: The other options are appropriate interventions, but the nurse should gather additional assessment data before contacting the physician for more orders.

A nurse is obtaining a history from a new client in the cardiovascular clinic. When investigating for childhood diseases and disorders associated with structural heart disease, which finding should the nurse consider significant? • Croup • Rheumatic fever • Severe staphylococcal infection • Medullary sponge kidney

*Rheumatic fever* Childhood diseases and disorders associated with structural heart disease include rheumatic fever and severe streptococcal (not staphylococcal) infections. Croup — a severe upper airway inflammation and obstruction that typically strikes children ages 3 months to 3 years — may cause latent complications, such as ear infection and pneumonia. However, it doesn't affect heart structures. Likewise, medullary sponge kidney, characterized by dilation of the renal pyramids and formation of cavities, clefts, and cysts in the renal medulla, may eventually lead to hypertension but doesn't damage heart structures.

The nurse is reviewing a patient's electrocardiogram (EKG) strip and notes the following: Heart rate of 68, regular rhythm, P waves normal in size and shape, PR interval 0.24 seconds and constant, QRS 0.08 seconds. What will the nurse document for the interpretation of the patient's EKG? Second-degree AV block type II Sinus rhythm with first-degree AV block Normal sinus rhythm Second-degree atrioventricular (AV) block type I

*Sinus rhythm with first-degree AV block* •The EKG shows what is best described as a sinus rhythm with a first-degree block. The rate is regular with a PR interval that is greater than 0.20 seconds and constant, and with QRS complexes 0.10 seconds or less. Incorrect options: •Second-degree AV block type I (also called Wenckebach or Mobitz type I) has more P waves than QRS complexes and the PR interval increases with each beat until a P wave appears without an accompanying QRS complex. •A normal sinus rhythm has a PR interval of 0.20 seconds or less. •In second-degree AV block type II (also called Mobitz type II), the PR intervals are prolonged but remain constant and the QRS complexes are usually wide.

While assessing a patient with an abdominal aortic aneurysm (AAA), the nurse should auscultate for bruits. Which location is the most common site for AAA formation? • Proximal to the renal arteries • Slightly distal to the aortic arch • Slightly distal to the renal arteries • Slightly distal to the iliac arteries•

*Slightly distal to the renal arteries* • 90% of AAAs occur distal to/below the renal arteries. • Incorrect: Aneurysms distal to the iliac artery are technically aneurysms of the iliac arteries, not of the aorta. • Incorrect: Suprarenal aortic aneurysms do occur, but are not common. • Incorrect: Debakey type III aortic dissection occurs in the descending aorta, just distal to the aortic arch (not abdominal).

Which would be most likely to assist the client with hypertension in maintaining an exercise program? • Give the client a written exercise program. • Explain the exercise program to the client's spouse. • Reassure the client that he or she can do the exercise program. • Tailor a program to the client's needs and abilities.

*Tailor a program to the client's needs and abilities.* Tailoring or individualizing a program to the client's lifestyle has been shown to be an effective strategy for changing health behaviors. Providing a written program, explaining the program to the client's spouse, and reassuring the client that he or she can do the program may be helpful but are not as likely to promote adherence as individualizing the program.

A visiting nurse is teaching a client with heart failure about taking his medications. The client requires six different medications that are taken at four different times per day. The client is confused about when to take each medication. How should the nurse intervene? • Come to the client's house each morning to prepare the daily allotment of medications. • Teach a family member to fill a medication compliance aid once per week so the client can independently take his medications. • Ask the physician if the client can take fewer pills each day. • Ask the client's family to take turns coming to the house at each administration time to assist the client with his medications.

*Teach a family member to fill a medication compliance aid once per week so the client can independently take his medications.* The nurse should intervene by asking a family member to fill a compliance aid each week with the client's weekly supply of medications in the appropriate time slots. Family members can't be expected to come to the client's house four times each day to administer medications. The physician shouldn't change the dosing regimen just for convenience. The home care nurse can't visit the client each morning to prepare the daily medication regimen

A nurse just received a shift report for a group of clients on the telemetry unit. Which client should the nurse assess first? • The client with a history of cardioversion for sustained ventricular tachycardia 2 days ago • The client admitted with first-degree atrioventricular (AV) block whose cardiac monitor now reveals type II second-degree AV block • The client with a history of heart failure who has bibasilar crackles and pitting edema in both feet • The client admitted for unstable angina who underwent percutaneous coronary intervention (PCI) with stenting yesterday

*The client admitted with first-degree atrioventricular (AV) block whose cardiac monitor now reveals type II second-degree AV block* The client whose cardiac rhythm now shows type II second-degree AV block should be assessed first. The client's rhythm has deteriorated from first-degree heart block to type II second-degree AV block and may continue to deteriorate into a lethal form of AV block (known as complete heart block). The client who underwent cardioversion 2 days ago has likely had the underlying reason for the sustained ventricular tachycardia corrected. The client with a history of heart failure may have chronic bibasilar crackles and pitting edema of both feet. Therefore, assessing this client first is not necessary. The client who underwent PCI with stenting was at risk for reperfusion arrhythmias and/or bleeding from the arterial puncture site but could be considered to be stable 24 hours post procedure.

A nurse is evaluating a client who had a myocardial infarction (MI) 7 days earlier. Which outcome indicates that the client is responding favorably to therapy? • The client demonstrates ability to tolerate more activity without chest pain. • The client exhibits a heart rate within normal limits. • The client requests information regarding smoking cessation. • The client is able to verbalize the action of all his prescribed medications.

*The client demonstrates ability to tolerate more activity without chest pain.* The ability to tolerate more activity without chest pain indicates a favorable response to therapy in a client who is recovering from an MI or who has a history of coronary artery disease. A heart rate within the normal limits of 60-100 per minute does not necessarily indicate a favorable response to treatment. Smoking is a cardiovascular risk factor that the client would be wise to eliminate, but it does not indicate favorable response to treatment. Knowledge of prescribed meds is a good thing, but again does not impact response to treatment.

A nurse administers the first dose of nadolol to a client with a blood pressure of 180/96. During an assessment, 4 hours later, which information indicates that the client needs immediate intervention? • The client's heart rate has decreased from 88 to 76 beats/minute. • The client's blood pressure (BP) is 142/90 mm Hg. • The client has cool fingers and toes bilaterally. • The client has wheezing throughout their lung fields.

*The client has wheezing throughout their lung fields.* Wheezing indicates the client is experiencing bronchospasms, which are a common adverse effect of a noncardioselective beta blocker. The nurse should immediately obtain an oxygen saturation measurement, apply supplemental oxygen, and notify the healthcare provider. The other symptoms are all expected effects of nadolol.

A client arrives in the emergency department reporting intense pain in the abdomen and tells the nurse that it feels like a heartbeat in the abdomen. Which nursing assessment would indicate potential rupture of an aortic aneurysm? • The client has been taking an antihypertensive for the past 3 years but forgot to take it today. • The blood pressure and pulse are within normal limits, but the client's skin color is pale and slightly diaphoretic. • The client reports feeling nauseated. • The client reports increasing severe back pain.

*The client reports increasing severe back pain.* Increased severe back pain and increased irritation to nerves are indicative of a potential rupture of an aneurysm. The client would be hypertensive and present with tachycardia, so the other choices are not correct. Nausea, although possible, or a missed dose of medication, do not indicate potential rupture.

A client is given amiodarone in the emergency department for a dysrhythmia. Which finding indicates the drug is having the desired effect? • The ventricular rate is increasing. • The absent pulse is now palpable. • The number of premature ventricular contractions is decreasing. • The fine ventricular fibrillation changes to coarse ventricular fibrillation.

*The number of premature ventricular contractions is decreasing.* Amiodarone is used for the treatment of premature ventricular contractions, ventricular tachycardia with a pulse, atrial fibrillation, and atrial flutter. Amiodarone is not used as initial therapy for a pulseless dysrhythmia.

A nurse working in the emergency department (ED) reviews arterial blood gas (ABG) values for a client diagnosed with heatstroke. Blood gas values are pH 7.48, pCO2 34, pO2 95, CO2 23, HCO3 22, and SO2 98%. What nursing interventions demonstrate the nurse's understanding of the patient's ABG's and knowledge of Maslow's hierarchy of needs when providing care for this patient? • The nurse prepares for endotracheal intubation and mechanical ventilation for the client • Lab values are within normal limits and contacts the client's family to be with the patient while in the ED • The nurse completes a spiritual assessment and provides appropriate clergy support for the client • The nurse immediately starts an intravenous line (IV) of dextrose 50% in a water solution (D50W)

*The nurse prepares for endotracheal intubation and mechanical ventilation for the client* This client is experiencing respiratory alkalosis related to heatstroke. The pH level is elevated in hyperventilation; the client's hyperventilation will "blow off" more CO2, leading to lower pCO2 levels. Decreased pCO2 is caused by hyperventilation. Decreased CO2 levels are seen in renal failure. Renal failure is a sign of heatstroke. With rapid breathing SO2 can be increased with deep or rapid breathing. Acute airway management is indicated to improve tissue oxygenation. Airway support meets the client's physiologic need for a clear airway. Spiritual support is a higher level (self-actualization) on Maslow's hierarchy. Providing IV management for circulatory support is a basic physiologic need; however, airway management is priority.

A nurse is monitoring the electrocardiogram (EKG) of a patient in the intensive care unit (ICU). The monitor shows normally appearing P waves and regular P to P interval and normal appearing QRS complexes with a regular R to R interval. The EKG demonstrates a P to R interval that is never constant indicating no correlation between any P waves and QRS complexes. The nurse knows that this indicates which of the following heart rhythms? Third-degree AV block Mobitz type II second-degree AV block Mobitz type I second-degree AV block First-degree atrioventricular (AV) block

*Third-degree AV block* •Third-degree AV heart block is seen when no atrial electrical impulses can conduct past the AV node to the ventricles resulting in regular P waves and regular QRS complexes; however, they have no relationship to each other and "march out" separately (see image). •All QRS complexes noted in third-degree AV block occur from intrinsic depolarization in the ventricles at a rate usually much less than 40 beats per minute. •Since there is no "communication" between the atria and ventricles, the upper and lower heart chambers beat independently of each other instead of being "in sync." •This condition will almost always require a pacemaker to prevent compromise or death from severely decreased cardiac output.

A nurse is at the bedside of a patient complaining of severe fatigue with increasing dyspnea. The nurse quickly reviews the patient's electrocardiogram (EKG) and sees a normal P-to-P interval and normal R-to-R interval. However, the PR intervals appear to be completely unrelated to the QRS complexes and the QT intervals are long. What condition does the nurse suspect the patient is experiencing? Third-degree heart block Ventricular fibrillation Junctional dysrhythmia Second-degree heart block

*Third-degree heart block* •*Third-degree heart block* (also known as complete heart block) occurs when there is no communication between the heart's ventricular & atrial conduction systems. •The P-to-P interval and R-to-R interval are regular because the rhythm of each chamber is regular; they just don't communicate with the other chamber. The P-R intervals vary because the P waves are not "in sync" with QRS. Finally, the QT interval is prolonged, causing a bradydysrhythmia. Incorrect options: •In a second-degree heart block, there are more P waves than QRS complexes and some P waves are not conducted. There is not a complete miscommunication between the atria and ventricles in this rhythm. •Ventricular fibrillation has an unobtainable rapid rate. The P-P and R-R intervals are also unobtainable, as is the PR interval. •With junctional dysrhythmias, there are abnormal P waves (either absent or inverted) and the P-to-P and R-to-R intervals are not regular.

A nurse is reviewing a patient's electrocardiogram (EKG) and notes that the T waves appear inverted. The nurses suspect which of the following as the most likely condition? Tissue ischemia Myocardial infarction Hypertension Hyperkalemia

*Tissue ischemia* •Inverted or depressed T waves on the patient's EKG indicate tissue ischemia or decreased oxygen to the tissues. Incorrect options: •Myocardial infarction typically manifests as changes to the ST segment of the patient's EKG, not the T wave. •Hypertension is a condition that cannot be directly seen on the patient's EKG. Because of this, changes to the T waves cannot be linked to hypertension. •Hyperkalemia is an elevated serum potassium level. This electrolyte imbalance typically causes peaked T waves, not inverted T waves.

The nurse is caring for a client post myocardial infarction (MI). Orders include strict bed rest and a clear, liquid diet. What is the nurse's best response to the client when inquiring about the purpose of the new diet? • To improve the gastric acidity of the stomach • To reduce the metabolic workload of digestion • To address the fluctuation in blood sugar • To reduce the amount of fecal elimination

*To reduce the metabolic workload of digestion* Acute care of the client with an MI is aimed at reducing the cardiac workload. Clear liquids are easily digested to help reduce this workload. Sympathetic nervous system involvement causes decreased peristalsis and gastric secretion, so limiting food intake helps prevent gastric distension and cardiac workload. A clear diet will not reduce gastric acidity or blood glucose, and fecal elimination will still occur, so these are incorrect choices.

A client has an International normalized ratio (INR) of 1.6, creatine kinase-MB (CK-MB) of 90 μ/L, troponin 2.1 ng/L, and myoglobin 90 μg/L. Which of the following results requires the nurse to take action? • INR of 1.6 • CK-MB of 90 μ/L • Troponin of 2.1 ng/L • Myoglobin 90 μg/L

*Troponin of 2.1 ng/L* Troponins I and T are cardiac enzymes that are only released when the cardiac muscle is damaged. Elevation of these values above the respective reference ranges of 0-0.1 ng/L or 0-0.2 ng/L indicates a myocardial infarction. Myoglobin is released when muscle cells are damaged. Myoglobin may rise above the normal level of 0-90 μg/L with a myocardial infarction (MI) but is not a clear indicator of MI because it can also rise during strenuous exercise, traumatic injury, and intramuscular injections. CK-MB will rise following MI, but may be elevated by events that also raise myoglobin. A normal range for CK-MB is between 30 and 170 μ/L. The INR test is a measure of blood clotting. An INR value of 1.6 is within the normal range.

A patient is brought to the emergency department with complaints of chest pain radiating to the shoulder and shortness of breath. Which of the following lab tests is most important to obtain and evaluate? Troponin-I Lactate dehydrogenase Creatine kinase (CK-MB) Myoglobin

*Troponin-I* • Troponin-I is highly specific for cardiac muscle necrosis and is the most accurate lab test for myocardial infarctions. It rises in 3-4 hours and can be detected for up to 10-14 days. Of note, an electrocardiogram (EKG) is also one of the first tests ordered for patients with a suspected myocardial infarction (MI). • Incorrect: CK-MB is used to test for cardiac damage but is also present in small amounts in skeletal muscle and it peaks in 12-24 hours, so it is not the best test to use. • Incorrect: The other tests are less specific than Troponin-I. Myoglobin is a protein that is present in both heart and skeletal muscles. Lactate dehydrogenase (LDH) is an enzyme found in almost all cells of the body, so this makes it the least specific of all the tests.

he nurse is reviewing laboratory results for a patient and finds a potassium level of 2.9. The nurse then looks at the patient's electrocardiogram (EKG) monitor and notes the occurrence of wide QRS waves at a rate of 100 with no identifiable P waves. The nurse immediately checks on the patient, who is awake and alert, but with complaints of shortness of breath and chest discomfort. The nurse calls the rapid response team for assistance. The nurse has recognized which of the following heart rhythms in this patient? Ventricular tachycardia Sinus bradycardia Atrial fibrillation Supraventricular tachycardia

*Ventricular tachycardia* Explanation •Wide QRS waveforms, with the absence of P waves, accompanied by an elevated rate at 100 indicates ventricular tachycardia. •A low potassium level can cause ventricular irritability, which may lead to ventricular arrhythmias. •In ventricular tachycardia, the patient can be awake and alert, but may have shortness of breath and chest pain. Patients in ventricular tachycardia may also be unconscious or lose consciousness during an episode of ventricular tachcardia (V-tach). •Ventricular tachycardia is an urgent or emergent situation.

While assessing a patient with aortic regurgitation, the nurse notes a Corrigan's pulse. This finding is characterized by • a weak pulse with a prolonged peak • a narrow pulse pressure • a weak, thready pulse • a bounding pulse, with a rapid rise and fall

*a bounding pulse, with a rapid rise and fall* • Corrigan's pulse is a bounding and forceful pulse with a rapid rise and sudden collapse. This is associated with increased stroke volume and a decrease in peripheral resistance. • Corrigan's pulse is commonly found in aortic regurgitation and often leads to a widening pulse pressure. • Incorrect: Pulsus tardus is a weak pulse with a slow upstroke and prolonged peak. It is seen in aortic stenosis. • Incorrect: A weak, thready pulse is a sign of low blood pressure or reduced cardiac output.

Which symptom should the nurse teach the client with unstable angina to report immediately to the health care provider (HCP)? • a change in the pattern of the chest pain • pain during sexual activity • pain during an argument • pain during or after a physical activity

*a change in the pattern of the chest pain* The client should report a change in the pattern of chest pain. It may indicate increasing severity of coronary artery disease. Pain occurring during stress or sexual activity would not be unexpected, and the client may be instructed to take nitroglycerin to prevent this pain. Pain during or after an activity such as lawn mowing also would not be unexpected; the client may be instructed to take nitroglycerin to prevent this pain or may be restricted from doing such activities.

The nurse is evaluating a 52-year-old male for risk factors for coronary artery disease (CAD). The patient is overweight, male, and smokes a pack a day. The nurse questions the patient about other risk factors including a history of atherosclerotic heart disease elevated HDL levels a history of diabetes a history of gout

*a history of diabetes* • The major risk factors for coronary heart disease are diabetes, smoking, increased low-density lipoprotein (LDL) levels, and hypertension. • Incorrect: Elevated high density liproprotein (HDL) levels are protective against CAD. • Incorrect: Atherosclerotic heart disease is just another name for coronary artery disease. • Incorrect: Gout is not associated with CAD

A patient recently had an implantable cardioverter-defibrillator (ICD) inserted. The patient expresses concern about the device discharging during sexual activities and shocking the patient's spouse. The nurse should explain that a shock may possibly be felt, but would not cause harm to the spouse an alarm will sound before each shock the shock will not be felt by the spouse physical contact may be dangerous and should be avoided

*a shock may possibly be felt, but would not cause harm to the spouse* • An ICD shock is delivered internally using much less power than external pads. It would not harm a person touching the patient because the charge is not high enough to cause harm, and it would be difficult to detect by anyone else. • Incorrect: An alarm will sound if the patient is near a magnetic field that may deactivate the device. • Incorrect: It is unnecessary for others to avoid contact with the patient.

Which finding is an indication of a complication of septic shock? • anaphylaxis • acute respiratory distress syndrome (ARDS) • chronic obstructive pulmonary disease (COPD) • mitral valve prolapse

*acute respiratory distress syndrome (ARDS)* ARDS is a complication associated with septic shock. ARDS causes respiratory failure and may lead to death, even after the client has recovered from shock. Anaphylaxis is a type of distributive or vasogenic shock. COPD is a functional category of pulmonary disease that consists of persistent obstruction of bronchial airflow and involves chronic bronchitis and chronic emphysema. Mitral valve prolapse is a condition in which the mitral valve is pushed back too far during ventricular contraction.

The nurse needs further education and training when the nurse describes Mobitz type I block as: an unstable rhythm that is usually caused by an ventricle muscle disturbance caused by a block in the atrioventricular (AV) node first-degree atrioventricular block the most severe form of heart block that carries a bad prognosis Wenckebach phenomenon a form of second-degree block

*an unstable rhythm that is usually caused by an ventricle muscle disturbance* *first-degree atrioventricular block *the most severe form of heart block that carries a bad prognosis* • Second-degree AV block is a problem with the electrical conduction system in the heart between the atria and the ventricles. • Mobitz type I heart block, also known as Wenckebach, involves prolonged PR intervals followed by blocked P waves (this results in a 'dropped' QRS complex). The cycle then repeats. No treatment is indicated unless the patient is experiencing symptoms, however recognizing this rhythm is important because treatment of conduction blocks is based on the type and degree of block. • Mobitz type I is the mildest form of second-degree heart block and carries a better prognosis compared to second degree Mobitz type II or third degree block (most severe). • Mobitz type I is a stable rhythm that usually does not result in clinical manifestations because the ventricular rate (heart rate) is typically adequate. • Mobitz type I is also known as the Wenckebach phenomenon in which the QRS complexes are typically "grouped" into twos, threes, fours, and so on. • Mobitz type I is a form of second-degree heart block. • Mobitz type I is caused by a block in the AV node.

A client with chronic arterial occlusive disease undergoes percutaneous transluminal coronary angioplasty (PTCA) for mechanical dilation of the right femoral artery. After the procedure, the client will require long-term administration of: • aspirin or acetaminophen. • pentoxifylline or acetaminophen. • aspirin or clopidogrel. • penicillin V or erythromycin.

*aspirin or clopidogrel.* After PTCA, the client begins long-term aspirin or clopidogrel therapy to prevent thromboembolism. Physicians order heparin for anticoagulation during this procedure; some physicians discharge clients with a prescription for long-term warfarin or low-molecular-weight heparin therapy. Pentoxifylline, a vasodilator used to treat chronic arterial occlusion, isn't required after PTCA because the procedure itself opens the vessel. The physician may order short-term acetaminophen therapy to manage fever or discomfort, but prolonged therapy isn't warranted. The client may need an antibiotic, such as penicillin or erythromycin, for a brief period to prevent infection associated with an invasive procedure; long-term therapy isn't necessary.

A nurse is caring for a post-operative patient after a mitral valve replacement. The nurse should monitor the patient's pulmonary capillary wedge pressure in order to assess the central venous pressure assess the right ventricular systolic pressure assess the cardiac output assess left ventricular end-diastolic pressure

*assess left ventricular end-diastolic pressure* • The mitral valve separates the left atrium from the left ventricle. • The pulmonary capillary wedge pressure (PCWP) provides an indirect measure of the left ventricular end-diastolic pressure (left ventricular preload). It is measured using a Swan-Ganz catheter inserted in a peripheral vein and ending in a branch of the pulmonary artery. • Clinically, the PCWP is used to measure left ventricular failure or the severity of mitral valve stenosis. Both conditions would elevate left arterial pressure which would elevate the PCWP. • When the PCWP is elevated, it is indicative of left ventricular heart failure. • After a mitral valve replacement, there would be concern that the reduction of regurgitated flow back into the left atrium with each beat may impair left ventricular function. • PCWP provides an indirect measurement for left atrial pressure (LAP). It does not measure central venous pressure, right ventricular systolic pressure, or cardiac output. Of note, the Swan-Ganz catheter can measure cardiac output, but a different method is used.

A client with a recent diagnosis of deep vein thrombosis (DVT) has sudden onset of shortness of breath and chest pain that increases with a deep breath. The nurse should first: • perform range of motion exercises in the involved leg. • call the health care provider (HCP). • administer morphine sulfate 2 mg IV. • assess the oxygen saturation.

*assess the oxygen saturation.* A client with deep vein thrombosis (DVT) is at high risk for a pulmonary embolism from an embolus traveling to the lung. Sudden onset of symptoms and worsening of chest pain with a deep breath suggest a pulmonary embolism. The nurse assesses the client and obtains oxygen saturation levels prior to calling the HCP and administering morphine. Range of motion is a preventive measure for DVT and is not appropriate that this time.

The nurse teaches a client who has recently been diagnosed with hypertension about following a low-calorie, low-fat, low-sodium diet. Which menu selection would best meet the client's needs? • mixed green salad with blue cheese dressing, crackers, and cold cuts • ham sandwich on rye bread and an orange • baked chicken, an apple, and a slice of white bread • hot dogs, baked beans, and celery and carrot sticks

*baked chicken, an apple, and a slice of white bread* Processed and cured meat products, such as cold cuts, ham, and hot dogs, are all high in both fat and sodium and should be avoided on a low-calorie, low-fat, low-salt diet. Dietary restrictions of all types are complex and difficult to implement with clients who are basically asymptomatic.

A client is in hypovolemic shock. To determine the effectiveness of fluid replacement therapy, the nurse should monitor the client's: • blood pressure. • hemoglobin level. • temperature. • heart rate.

*blood pressure.* With adequate fluid replacement, fluid volume in the intravascular space expands, raising the client's blood pressure. The hemoglobin level reflects red blood cell concentration, not overall fluid status. Temperature and heart rate aren't directly related to fluid status. Remediation:

The nurse is assessing a patient with arterial insufficiency. The patient complains of lower leg pain when walking up the stairs, but states that it is relieved by rest. This phenomenon is called circulation after failed attempts at resuscitation. Raynaud's phenomenon vasospasms Lazarus syndrome claudication

*claudication* • Claudication refers to a cramp-like pain in the leg muscles after a small amount of physical activity. It is caused by poor circulation to the extremities because the muscles in the legs (usually the calves) are not getting adequate oxygen to meet increased demands during periods of activity. • Incorrect: Raynaud's phenomenon is a vasospastic disorder that compromises blood flow to the extremities and is triggered by cold temperatures or stress, not physical activity. • Incorrect: Vasospasm refers to sudden, involuntary constriction of blood vessels and is not related to physical activity. • Incorrect: Lazarus syndrome is the spontaneous return of circulation after failed attempts at resuscitation.

The nurse has completed an assessment on a client with a decreased cardiac output. Which findings should receive the highest priority? • BP 110/62 mm Hg, atrial fibrillation with HR 82 bpm, bilateral basilar crackles • confusion, urine output 15 mL over the last 2 hours, orthopnea • SpO2 92% on 2 L nasal cannula, respirations 20 breaths/min, 1+ edema of lower extremities • weight gain of 1 kg in 3 days, BP 130/80 mm Hg, mild dyspnea with exercise

*confusion, urine output 15 mL over the last 2 hours, orthopnea* A low urine output and confusion are signs of decreased tissue perfusion. Orthopnea is a sign of left-sided heart failure. Crackles, edema, and weight gain should be monitored closely, but the levels are not as high a priority. With atrial fibrillation, there is a loss of atrial kick, but the blood pressure and heart rate are stable.

A patient with Raynaud's disease is admitted to the hospital. The nurse assesses the patient for which expected clinical manifestations? *Select all that apply.* diminished or absent peripheral pulses urticaria of the upper extremities thickening of the skin on the hands and feet brittle finger nails numbness in the hands pale extremities

*diminished or absent peripheral pulses* *brittle finger nails* *numbness in the hands* *pale extremities* • Raynaud's disease (or phenomenon) is a vasospastic disorder affecting the hands and feet. It leads to impaired perfusion. An attack usually only lasts minutes to hours. • Signs and symptoms include diminished peripheral pulses, pallor, numbness in the extremities, brittle finger nails, thin skin, and cyanosis. • Vasospasm generally occurs due to exposure to cold or stress. • Urticaria and thickening of the skin is not associated with Raynaud's disease.

Automated external defibrillators (AEDs) are used in cardiac arrest situations for: • early defibrillation in cases of atrial fibrillation. • cardioversion in cases of atrial fibrillation. • pacemaker placement. • early defibrillation in cases of ventricular fibrillation.

*early defibrillation in cases of ventricular fibrillation.* AEDs are used for early defibrillation in cases of ventricular fibrillation. The American Heart Association and Canadian Heart and Stroke Foundation place major emphasis on early defibrillation for ventricular fibrillation and use of the AED as a tool to ↑ sudden cardiac arrest survival rates.

A client with heart failure has bilateral +4 edema of the right ankle that extends up to midcalf. The client is sitting in a chair in no evident distress with the legs in a dependent position. The nurse should first: • assist the client to bed • request a prescription for support stockings • elevate the client's legs on a foot stool • take the client's blood pressure

*elevate the client's legs on a foot stool* Decreasing venous congestion in the extremities is a desired outcome for clients with heart failure. The nurse should elevate the client's legs. It is not necessary for the client to return to bed. Support stockings are not indicated at this time. The client is not having difficulty breathing or other signs of distress; it is not necessary to take the vital signs.

The nurse is assessing the hepatojugular reflux in a patient with congestive heart failure. To perform this assessment technique correctly, the nurse should lie the patient flat compress the abdomen for 5-15 seconds elevate the head of bed to 45 degrees press the left upper quadrant of the abdomen

*elevate the head of bed to 45 degrees* • The hepatojugular reflux test is used to measure jugular vein distention seen in tricuspid regurgitation, heart failure, and other conditions. • The nurse places the patient's head at 45 degrees. • The nurse observes the jugular pulsations during quiet respirations, then firmly presses the right upper quadrant or center of the abdomen for 30-60 seconds. • The nurse then again observes the jugular pulsations during respirations. If there is a sustained rise in the jugular vein (>3 cm), the result is positive.

A patient returns to the nursing unit after undergoing a cardiac catheterization using the femoral insertion site. After the procedure, the nurse should avoid • assessing the motor function of the patient's foot on the affected side • elevating the head of the bed • providing oral fluids • resuming all medications

*elevating the head of the bed* • After a cardiac catheterization using the femoral artery, the patient should remain on flat bedrest and be reminded not to flex or move the affected extremity after the sheath removal. Doing so may result in hemorrhage or arterial occlusion. The site is monitored for bleeding or hematoma and kept immobilized for a few hours. • Incorrect: Fluids are encouraged after any procedure that uses contrast dye. • Incorrect: Assessing the motor function of the patient's foot is important to confirm adequate perfusion. • Incorrect: Medications should be resumed to treat all other diseases and conditions. assessing the motor function of the patient's foot on the affected side

A fourth heart sound (S4) indicates a: • dilated aorta. • normally functioning heart. • decreased myocardial contractility. • failure of the ventricle to eject all blood during systole.

*failure of the ventricle to eject all blood during systole.* An S4 occurs as a result of ↑ resistance to ventricular filling following atrial contraction. This ↑ resistance is related to decreased ventricular compliance. A dilated aorta doesn't cause an extra heart sound, though it does cause a murmur. A nurse hears decreased myocardial contractility as a third heart sound. She doesn't hear an S4 in a normally functioning heart.

A client who has been experiencing angina has a new prescription for nitroglycerin. The nurse should instruct the client to report having which potential side effect of nitroglycerin? • headache • shortness of breath • bradycardia • hypertension

*headache* The most common side effect of nitroglycerin is a headache. Additional cardiovascular side effects include tachycardia, hypotension, and dizziness. Nitroglycerin does not cause shortness of breath, bradycardia, or hypertension.

The nurse is educating a patient with myocarditis. The nurse explains to the patient that myocarditis affects the outermost layer of the heart heart muscle inner layer of the heart heart valves

*heart muscle* • Myocarditis is inflammation of the myocardium (the muscle of the heart). • Myocarditis is most often caused by viral infections but may also be caused by an autoimmune reaction to drugs or pathogens. • Incorrect: Pericarditis is inflammation of the outermost layer of the heart. • Incorrect: Endocarditis is inflammation of the innermost layer of the heart. • Incorrect: Valvular disease affects the heart valves.

The nurse monitors the serum electrolyte levels of a client who is taking digoxin. Which electrolyte imbalance is a common cause of digoxin toxicity? • hyponatremia • hypomagnesemia • hypocalcemia • hypokalemia

*hypokalemia* Hypokalemia is one of the most common causes of digoxin toxicity. It is essential that the nurse carefully monitor the potassium levels of clients taking digoxin to avoid toxicity. Low serum potassium levels can cause cardiac dysrhythmias.

The nurse is encouraging a patient with peripheral artery disease (PAD) to start a progressive exercise program. The nurse explains that exercise will improve symptoms of intermittent claudication because exercise will reduce body weight promote venous return increase high-density lipoproteins (HDLs) improve collateral circulation

*improve collateral circulation* • PAD results from compromised blood flow to the extremities. While intermittent claudication occurs during activities such as walking, a progressive exercise program improves circulation and collateral blood flow around the affected (blocked) arteries. • Exercise programs should be progressive. The patient should start with walking or other mild exercises and increase intensity as tolerated. • Incorrect: Although exercise will increase HDLs, reduce body weight, and promote venous return, these changes do not directly improve symptoms of PAD.

A nurse knows that the major clinical use of dobutamine is to: • increase cardiac output. • prevent sinus bradycardia. • treat hypotension. • treat hypertension.

*increase cardiac output.* Dobutamine ↑ cardiac output for clients with acute heart failure and those undergoing cardiopulmonary bypass surgery. Physicians may use epinephrine hydrochloride, another catecholamine agent, to treat sinus bradycardia. Physicians use many of the catecholamine agents, including epinephrine, isoproterenol, and norepinephrine, to treat acute hypotension. They don't use catecholamine agents to treat hypertension because catecholamine agents may raise blood pressure.

A patient with Raynaud's disease is admitted to the hospital with cyanotic fingers and extreme pain. The nurse understands that these symptoms are associated with prolonged bed rest ingestion of caffeinated beverages ingestion of alcohol exposure to extreme heat

*ingestion of caffeinated* Raynaud's disease is characterized by intermittent vasospastic attacks. These acute attacks are often precipitated by stress, cold temperatures, caffeine, or smoking. • Incorrect: Prolonged bed rest is not associated with Raynaud's disease. • Incorrect: Alcohol and extreme heat cause vasodilation, not vasospasm.

The nurse is assessing a patient in the emergency room due to chest pain. To determine whether the patient's pain is due to a myocardial infarction, the nurse should note the pain is described as gnawing is accompanied by nausea and vomiting is unrelieved by nitroglycerin, but is relieved by morphine began with physical activity but resolved with rest

*is unrelieved by nitroglycerin, but is relieved by morphine* • Pain from a myocardial infarction (MI) usually lasts longer than 30 minutes, radiates to the left arm and shoulder, and requires morphine for relief because myocardial tissue has died from hypoxia. The onset of chest pain from an MI can begin with activity or exertion, but does not improve with rest or use of nitroglycerin. • Incorrect: Anginal pain is usually caused by activity or exertion and is relieved with rest or use of nitroglycerin as perfusion is improved with vasodilation. • Incorrect: Gnawing or burning pain is usually related to a gastrointestinal (GI) disorder. MI chest pain is usually described as pressure, tightness, or squeezing. • Incorrect: Nausea and vomiting may accompany a myocardial infarction, but can be present with many other disorders.

The nurse is caring for a patient complaining of chest pain related to pericarditis. To help relieve the pain, the nurse should instruct the patient to lie in the supine position lie in the dorsal recumbent position lean forward while sitting lie in the prone position

*lean forward while sitting* • Pericarditis is inflammation of the pericardium caused by infections, autoimmune disorders, or trauma. • Instructing the patient to lean forward will pull the heart away from the lungs, preventing pericardial irritation caused by friction and contact with the lungs. • Incorrect: Pain related to pericarditis worsens while in any lying position.

The correct landmark for obtaining an apical pulse is the: • left fifth intercostal space, midaxillary line. • left fifth intercostal space, midclavicular line. • left second intercostal space, midclavicular line. • left seventh intercostal space, midclavicular line.

*left fifth intercostal space, midclavicular line.* The correct landmark for obtaining an apical pulse is the left fifth intercostal space in the midclavicular line. This area is the point of maximum impulse and the location of the left ventricular apex. The left second intercostal space in the midclavicular line is where the nurse auscultates pulmonic sounds. The apical pulse isn't obtained at the midaxillary line or the seventh intercostal space in the midclavicular line.

The nurse is coaching a client with heart failure about reducing fluid retention. Which strategy will be most effective in reducing a client's fluid retention? • low-sodium diet • walking for 20 minutes 3 times a week • restricting fluid intake • elevating the feet

*low-sodium diet* In clients with fluid retention, sodium restriction may be necessary to promote fluid loss. Increasing exercise will not reduce fluid retention. Exercise will promote circulation, but will not manage the fluid retention. Restricting fluid intake will not reduce retained fluids; ↑ fluids will ↑ urine output and promote improved fluid balance. Elevating the client's feet helps promote venous return and fluid reabsorption but in itself will not reduce the volume of excess fluid.

The nurse is preparing the client with heart failure to go home. The nurse should instruct the client to: • monitor urine output daily. • maintain bed rest for at least 1 week. • monitor daily potassium intake. • monitor weight daily.

*monitor weight daily.* People with heart failure are taught to maintain a target weight and to weigh themselves daily to monitor increasing fluid retention. Fluid retention can lead to decompensation and hospitalization. Monitoring daily urine output is not required of these clients. A week of bed rest is not indicated for most people with heart failure. Clients on potassium-wasting diuretics will be taught to include dietary sources of potassium or to take a potassium supplement. However, all clients with heart failure should weigh themselves daily to monitor fluid status.

The nurse is caring for a patient with pericarditis. Knowing that this condition places the patient at risk for cardiac tamponade, the nurse should monitor for bounding heart sounds hypertension muffled heart sounds bradycardia

*muffled heart sounds*

A client has been diagnosed with peripheral arterial occlusive disease. In order to promote circulation to the extremities, the nurse should instruct the client to: keep the extremities elevated slightly. participate in a regular walking program. use a heating pad to promote warmth. massage calf muscles if pain occurs.

*participate in a regular walking program.* Clients diagnosed with peripheral arterial occlusive disease should be encouraged to participate in a regular walking program to help develop collateral circulation. They should be advised to rest if pain develops and resume activity when pain subsides. With arterial disease, extremities should be kept in a dependent position to promote circulation; elevation of the extremities will decrease circulation. To avoid burns, heating pads should not be used by anyone with impaired circulation. Massaging the calf muscles will not decrease pain. Intermittent claudication subsides with rest.

A floor nurse is receiving report on a patient being transferred from the intensive care unit (ICU). The ICU nurse reports that the patient's pulmonary artery wedge pressures are high. The floor nurse should expect to note Select all that apply. polyuria respiratory alkalosis pulmonary crackles. hypertension

*pulmonary crackles* *hypertension* • An increase in pulmonary artery wedge pressure indicates an increase in fluid and pressure in the pulmonary circulation, leading to pulmonary edema and crackles. This is indicative of left-sided heart failure. • Hypervolemia is another cause for high wedge pressures and hypertension would be expected if this were the cause. • Incorrect: Polyuria is unlikely because heart failure results in a decrease in perfusion and low urine output. • Incorrect: Acidosis is more likely with heart failure.

What is a priority nursing assessment in the first 24 hours after admission of the client with a thrombotic stroke? • cholesterol level • pupil size and pupillary response • bowel sounds • echocardiogram

*pupil size and pupillary response* It is crucial to monitor the pupil size and pupillary response to indicate changes around the cranial nerves. The cholesterol level is not a priority assessment, although it may be an assessment to be addressed for long-term healthy lifestyle rehabilitation. Bowel sounds need to be assessed because an ileus or constipation can develop, but this is not a priority in the first 24 hours, when the primary concerns are cerebral hemorrhage and ↑ intracranial pressure. An echocardiogram is not needed for the client with a thrombotic stroke without heart problems.

While caring for a patient with severe heart failure and coronary artery disease, the nurse understands that her patient's decreased preload could be caused by fluid overload sepsis or hemorrhage heart rate of 55 heart failure

*sepsis or hemorrhage* • Preload is determined by how much the muscles of the heart stretch due to the blood volume that has filled the ventricles at the end of diastole. Preload is decreased by conditions that reduce circulating volume or venous return. This includes hemorrhage, sepsis, and anaphylaxis. • Incorrect: Fluid overload and heart failure increase the circulating volume and preload. • Incorrect: A slower heart rate increases filling time which increases the amount of blood that fills the ventricles and increases preload.

An obese diabetic client has bilateral leg aching is to start a cardiac rehabilitation with an exercise program. Using which exercise equipment will be most helpful to the client? • elliptical trainer • treadmill • stair climber • stationary bicycle

*stationary bicycle* The stationary bicycle is the most appropriate training modality because it is a non-weight-bearing exercise. The time that the individual exercises on the stationary bicycle is increased with improved functional capacity. The other exercise equipment requires exercising while standing.

The nurse is reviewing the electrocardiogram (EKG) strip of a patient with wide QRS complexes. The nurse understands that the QRS complex represents Purkinje fiber depolarization ventricular depolarization atrial repolarization ventricular repolarization

*ventricular repolarization* • Ventricular depolarization is represented by the QRS complex. • Ventricular repolarization is represented by the T wave. • Incorrect: Atrial repolarization is hidden within the QRS complex and cannot be distinguished. • Incorrect: Purkinje fiber depolarization is not represented on the EKG.

A client had a repair of a thoracoabdominal aneurysm 2 days ago. Which finding should the nurse consider unexpected and report to the health care provider (HCP) immediately? • adominal pain at 5 on a scale of 0 to 10 for the last 2 days • heart rate of 100 beats per minute after ambulating 200 feet • (0.06 km) urine output of 2,000 mL in 24 hours • weakness and numbness in the lower extremities

*weakness and numbness in the lower extremities* One of the complications of a thoracoabdominal aneurysm repair is spinal cord injury. Therefore, it is important for the nurse to assess for signs and symptoms of neurologic changes at and below the site where the aneurysm was repaired. The client is expected to have moderate pain following surgery. An elevated heart rate is expected after physical exertion. It is important to monitor urine output following aneurysm surgery, but a urine output of 2,000 mL in 24 hours is adequate following surgery.

A client with Raynaud's phenomenon is considering having a sympathectomy. This nurse should tell the client that the surgery is performed: • in the early stages of the disease to prevent further circulatory disturbances. • when the disease is controlled by medication. • when all other treatment alternatives have failed. • when the client is unable to control stress-related vasospasm.

*when all other treatment alternatives have failed.* Sympathectomy is scheduled only after other treatment alternatives have been explored and have failed. Medication and stress management are beneficial strategies to prevent advancement of the disease process. If the disease is controlled by medication, there is no reason for surgery.

The nurse is caring for a patient at risk for premature ventricular contractions (PVCs). While monitoring the patient's electrocardiogram (EKG) rhythm, the nurse should look for wide QRS complexes in the presence of inverted P waves wide QRS complexes in the absence of P waves peaked P waves with a narrow QRS complex narrow QRS complexes

*wide QRS complexes in the absence of P waves* • PVCs are wide, bizarre QRS complexes in the absence of P waves, usually followed by a fully-compensated pause. • PVCs may increase with anxiety, stress, use of alcohol, stimulant medication, hypoxia, or electrolyte abnormalities. • PVCs are caused by abnormal automaticity of ventricular cells and are very common. They may occur erratically or at regular intervals. Trigeminy is every third beat, bigeminy is every second beat. • Narrow QRS complexes do not represent PVCs, the QRS complex is instead wide. • PVCs occur without a P wave before them as the ventricles contract before the atria.

Nurses can expect which of the following interventions to help regulate or treat patients with atrial flutter? Select all that apply. Administration of a non-dihydropyridine calcium channel blocker or a beta blocker as ordered Administration of heparin as ordered Administration of ibutilide intravenously as ordered Sternal massage Cardioversion Preparation of the client for radiofrequency catheter ablation

*• Administration of a non-dihydropyridine calcium channel blocker or a beta blocker as ordered* *• Administration of heparin as ordered* *• Administration of ibutilide intravenously as ordered* *• Cardioversion* *• Preparation of the client for radiofrequency catheter ablation* • Atrial flutter is an abnormal cardiac rhythm characterized by rapid, regular atrial depolarizations at a characteristic rate of approximately 300 beats per minute and a regular ventricular rate of about 150 beats per minute in patients not taking atrioventricular (AV) nodal blockers. • It can lead to symptoms of palpitations, shortness of breath, fatigue, or lightheadedness, as well as an increased risk of atrial thrombus formation that may travel elsewhere, causing cerebral or systemic emboli. • Administration of a non-dihydropyridine calcium channel blocker or a beta blocker will help reduce heart rate. • Anticoagulation is imperative to prevent systemic embolization, initially with heparin in clients with atrial flutter. • Intravenous ibutilide, a short-acting antidysrhythmic medication, has a 60% to 90% success rate for converting atrial flutter. • Cardioversion promptly restores sinus rhythm in atrial flutter, usually at energy levels as low as 25 to 100 joules. • Due to the high rate of recurrence of atrial flutter in patients without a correctable cause, and because of its high success rate, radiofrequency catheter ablation is the definitive treatment. • Incorrect: Sternal massage is not a treatment for atrial flutter.

The nurse is caring for an older adult with mild dementia admitted with heart failure. What nursing care will be helpful for this client in reducing potential confusion related to hospitalization and change in routine? Select all that apply. • Reorient frequently to time, place and situation. • Put the client in a quiet room furthest from the nursing station. • Perform necessary procedures quickly. • Arrange for familiar pictures or special items at bedside. • Limit the client's visitors. • Spend time with the client, establishing a trusting relationship.

*• Reorient frequently to time, place and situation.* * • Arrange for familiar pictures or special items at bedside.* *• Spend time with the client, establishing a trusting relationship.* It is not unusual for the elderly client to become somewhat confused when "relocated" to the hospital, and this may be more difficult for those with known dementia. Frequent reorientation delivered patiently and calmly along with placing familiar items nearby so the client can see them may help decrease confusion related to hospitalization. Establishing a trusting relationship is important with every client but maybe more so with this client. Putting the client in a room further from the nursing station may decrease extra noise for the client, but will also make it more difficult to observe the client and maintain a safe environment. Procedures should be explained to the client prior to proceeding and should not be rushed. Visits by family and friends may help to keep the client oriented.

A client with hypertensive emergency is being treated with sodium nitroprusside. In a dilution of 50 mg/250 mL, how many micrograms of nitroprusside are in each milliliter? Record your answer using a whole number.

200 First, calculate the number of milligrams per milliliter: 50 mg/250 mL = 1 mg/5 mL = 0.2 mg/1 mL Next, calculate the number of micrograms in each milligram: 0.2 mg × 1,000 mcg = 200 mcg.

A client with heart failure is receiving furosemide, 40 mg I.V. The physician orders [40 mEq (40 mmol/L)] of potassium chloride in 100 ml of dextrose 5% in water to infuse over 4 hours. The client's most recent serum potassium level is [3.0 mEq/L (3.0 mmol/L)]. At what infusion rate should the nurse set the I.V. pump?

25 ml/hour

The rapid response team has been called to manage an unwitnessed cardiac arrest in a client's hospital room. The estimated maximum time a person can be without cardiopulmonary function and still not experience permanent brain damage is: • 1 to 2 minutes. • 4 to 6 minutes. • 8 to 10 minutes. • 12 to 15 minutes. After a person is without cardiopulmonary function for 4 to 6 minutes, permanent brain damage is almost certain. To prevent permanent brain damage, it is important to begin CPR promptly after a cardiopulmonary arrest.

4 to 6 minutes. After a person is without cardiopulmonary function for 4 to 6 minutes, permanent brain damage is almost certain. To prevent permanent brain damage, it is important to begin CPR promptly after a cardiopulmonary arrest.

A nurse on the telemetry unit is faced with various monitor rhythms. Which rhythm takes priority? • A client's cardiac monitor suddenly reveals sinus tachycardia with isolated premature ventricular contractions. • A client's cardiac rhythm suddenly changes from normal sinus rhythm to uncontrolled atrial fibrillation. • A client's monitor shows frequent paced beats with capture. • A client's monitor shows sinus tachycardia with frequent premature atrial contractions (PACs).

A client's cardiac rhythm suddenly changes from normal sinus rhythm to uncontrolled atrial fibrillation. The client whose cardiac rhythm suddenly changes from normal sinus rhythm to uncontrolled atrial fibrillation takes priority. This cardiac rhythm change may cause clots to shower from the atria, placing the client at risk for a stroke. The client whose cardiac monitor reveals sinustachycardia with isolated premature ventricular contractions is not experiencing a life-threatening situation; therefore, he does not take priority. Frequent paced beats with capture is a normal finding for a client with a pacemaker. Sinus tachycardia with premature atrial contractions is not a priority situation.

The nurse is evaluating a patient with decreased cardiac output related to a myocardial infarction. Which of the following indicates improved cardiac output? Absence of angina and dyspnea with activity Heart rate of 54 beats per minute Blood pressure of 90/50 Cardiac output of 2.5 liters per minute

Absence of angina and dyspnea with activity • Decreased cardiac output can cause angina and dyspnea with activity. Improvement of cardiac output should resolve these symptoms. • Incorrect: A low heart rate or blood pressure may indicate poor cardiac output. • Incorrect: A cardiac output of 2.5 L per minute is not sufficient. Normal cardiac output is 4-8 L per minute.

· Either unstable angina or MI · Plaque ruptures results in clot formation and vasoconstriction

Acute Coronary Syndromes

A nurse is caring for a client with moderate acute decompensated heart failure. The nurse should include which nursing intervention in the care plan? Select all that apply. Administer furosemide as ordered. Administer dobutamine as ordered. Reduce stress. Elevate the legs. Limit fluid intake to 500 ml/day.

Administer furosemide as ordered. Administer dobutamine as ordered. Reduce stress. • First-line therapy generally includes a loop diuretic such as furosemide, which will inhibit sodium chloride reabsorption in the ascending loop of Henle. Diuretics reduce circulating blood volume, diminish preload, and lessen systemic and pulmonary congestion. • The most powerful way to increase contractility of the heart is to use inotropic medications. Principal inotropic agents include dobutamine, milrinone, dopexamine, and digoxin. • Aside from improving ventricular pump performance and reducing myocardial workload, the client also needs to reduce physical and emotional stress. • Even though the legs are edematous, they should not be elevated. Elevating the legs rapidly increases venous return to the heart. • It is not necessary to restrict fluid intake in clients with mild or moderate heart failure. In more advanced cases, however, it is beneficial to limit water to 1000 mL/day.

A client admitted with a massive myocardial infarction rapidly develops cardiogenic shock. Ideally, the physician would use the intra-aortic balloon pump (IABP) to support the injured myocardium. However, this client has a history of unstable angina pectoris, aortic insufficiency, hypertension, and diabetes mellitus. Which condition is a contraindication for IABP use? • Unstable angina pectoris • Diabetes mellitus • Hypertension • Aortic insufficiency

Aortic insufficiency A history of aortic insufficiency contraindicates use of the IABP. Other contraindications for this therapy include aortic aneurysm, central or peripheral atherosclerosis, chronic end-stage heart disease, multisystemic failure, chronic debilitating disease, bleeding disorders, and a history of emboli. Unstable angina pectoris that doesn't respond to drug therapy is an indication for IABP, not a contraindication. Hypertension and diabetes mellitus aren't contraindications for IABP.

A nurse places electrodes on a collapsed individual who was visiting a hospitalized family member, the monitor exhibits the following. Which interventions would the nurse do first? • Place the client on oxygen. • Confirm the rhythm with a 12-lead ECG. • Administer amiodarone (Cordarone) IV as prescribed. • Assess the client's airway, breathing, and circulation.

Assess the client's airway, breathing, and circulation. The rhythm the client is experiencing is ventricular tachycardia (VT). Although all of the options listed are appropriate for someone with stable VT, it is not yet known whether the client's VT is stable, unstable, or pulseless. Therefore, the nurse must first assess the airway, breathing, circulation, and level of consciousness to establish the client's stability. Different actions are required if the client's VT is unstable or pulseless.

- Initiate the impulse from the SA node - Pacing function, spontaneous and repetitive - Pacemaker cells - Do not need outside influence to generate - 60-100 bpm

Automaticity

Beat, PVC, Beat, PVC (every 2nd beat)

Bigeminy PVC

When caring for a patient with mitral valve stenosis, the nurse should monitor for which of the following symptoms associated with this condition? *Select all that apply.* Respiratory alkalosis Hypoventilation Blood-tinged sputum Fatigue Edema in the legs

Blood-tinged sputum Fatigue Edema in the legs • Mitral stenosis is a narrowing of the mitral orifice and thickening of the mitral valve leaflets, which impedes blood flow into the left ventricle. Blood picks up oxygen in the lungs and re-enters the heart into the left atrium, then must pass through the narrowed and thickened mitral valve into the left ventricle of the heart. The left ventricle is the main chamber, which pumps oxygen-rich blood to the body through the aorta. • As blood strains to pass through the mitral valve, pressures rise in the left atrium and then in the lungs, where the blood meets vascular resistance as it backs up. This causes pulmonary congestion and shortness of breath, which is worse with activity (as oxygen needs increase). • If untreated, the pressure and fluid buildup in the lungs increases the workload of the right side of the heart, which has to pump harder to send blood into the lungs to be oxygenated. Over time, right-sided heart failure develops. Complications include secondary pulmonary hypertension, atrial fibrillation, and thrombus. • Symptoms associated with mitral stenosis include fatigue (especially with activity), edema in the arms and legs (as right-sided heart failure develops), coughing (sputum may be blood- tinged), chest pain, and palpitations. A third heart sound may be heard on auscultation, and patients often have a low oxygen saturation even with supplemental 02. it is diagnosed by echocardiography. • The most common cause of mitral valve stenosis is rheumatic heart disease caused by rheumatic fever, which can develop after a strep infection (strep throat). • Incorrect: Fluid overload of the pulmonary circulation can impair gas exchange, potentially leading to respiratory acidosis as CO2 builds up. • Incorrect: Hypoventilation refers to slow breathing. Patients with mitral stenosis have shortness of breath from pulmonary congestion, which often contributes to faster breathing to compensate.

All normal except QRS complex widened Treatment: usually none

Bundle Branch Block (BBB)

- Connects with the base of the AV node - Goes down into the septum and then splits into right and left bundle branches that move around the ventricles

Bundle of His

95% of sudden cardiac arrest die before reaching hospital Mainly because they are in vfib AED's help ↓ risk factors Lower lipid levels/add Omega-3 from fish

CAD Health Promotion and Maintenance

ECHO Stress test Cardiac cath

CAD Imaging

Restore perfusion limits the damage to the ventricles --4-6 hrs for NSTEMI --60-90 min for STEMI ASA 162-325 mg immediately and daily unless contraindicated MONA- morphine (vasodilate), oxygen, nitro (vasodilate), aspirin (thins blood) BetaBlocker - decrease the size of the infarct, ↓ dysrhythmias and ↓mortality rates Open the artery

CAD Interventions

Troponins CK-MB

CAD Labs

Meds ECG within 10 min of pain O2 Vitals ABC's

CAD Tx

The nurse instructs a client with coronary artery disease in the proper use of nitroglycerin. The client has had 2 previous episodes of coronary artery disease. At the onset of chest pain, what should the client do? • Call 911 when three nitroglycerin tablets taken every 5 minutes are not effective. • Call 911 when five nitroglycerin tablets taken every 5 minutes are not effective. • Go to the emergency department if two nitroglycerin tablets taken 5 minutes apart are not effective. • Take one tablet and then immediately call 911.

Call 911 when three nitroglycerin tablets taken every 5 minutes are not effective. Nitroglycerin tablets should be taken 5 minutes apart for three doses; if this is ineffective, 911 should be called to obtain an ambulance to take the client to the emergency department. The client should not drive or have a family member drive the client to the hospital.¬

*Phase 1* Acute illness ends with d/c from hospital Promote rest Assist with ADL's *Phase 2* Begins after d/c, cont with convalescence at home Begin a program for cardiac rehab *Phase 3* Long term conditioning

Cardiac rehab

*Nonsurgical* • Same as for HF • Antidysrhythmics • ICD's • BetaBlockers *Surgical* • Ablation • Transplant

Cardiomyopathy *Interventions*

A nurse is preparing a teaching plan for a male client newly prescribed atenolol. Which information is important for the nurse to teach this client? • Causes and treatments for erectile dysfunction • Control of excessive flatus • Management of incontinence • Prevention of constipation

Causes and treatments for erectile dysfunction Erectile dysfunction is a potential adverse effect of beta blockers. The other symptoms are not side effects of this drug.

The nurse is caring for a post-op client after renal artery bypass for an arterial occlusion. To promote the goal of effective kidney function, the nurse includes which intervention(s) in the plan of care? Select all that apply. Check the client's dressings for excessive drainage Monitor creatinine levels Maintain strict inputs & outputs (I&O) Monitor urine specific gravity Administer IV fluids as ordered Evaluate the effectiveness of pain medication after each administration

Check the client's dressings for excessive drainage Monitor creatinine levels Maintain strict inputs & outputs (I&O) Monitor urine specific gravity Administer IV fluids as ordered • By definition, Acute Kidney Injury (AKI) is an abrupt decrease (within 48 hours) in kidney function that includes an increase in serum creatinine (either an absolute increase of greater than 0.3 mg/dL or a percentage increase of 50%) or a decrease in urine output (less than 0.5 mL/kg/hour for more than 6 hours). It may be caused by disease or injury. Kidney function usually returns to baseline if AKI is identified early and appropriately treated. • To promote effective kidney function, the nurse should include interventions that support renal perfusion, monitor the patient's kidney function by evaluating labs and data being collected, and reporting noteworthy findings. • Administering IV fluids ensures adequate fluid volume for circulation of oxygenated blood to the renal and peripheral tissues. • Maintaining strict I&O promotes the goal of effective tissue perfusion by collecting data on the progression and treatment of the acute kidney injury. • Monitoring serial creatinine levels evaluates kidney function following response to treatment. • Monitoring urine specific gravity is important. This measures the kidney's ability to concentrate urine. In AKI, specific gravity is usually equal to or less than 1.010, indicating loss of ability to concentrate the urine. If urine specific gravity trends lower in the recovery phase, this requires reporting to the provider to facilitate evaluation and intervention. • Blood and drainage may appear on dressings or beneath the site because of gravity. Monitoring for this is important, and it contributes toward the goal of kidney function by monitoring for complications that would compromise renal perfusion. • Remember that "interventions" may include monitoring, evaluating, or observing. These are interventions when implemented to address patient problems already identified in the assessment phase. Incorrect options: • Evaluating the effectiveness of pain medication helps determine the adequacy of analgesia, but does not address kidney function directly.

· *angina pectoris* Chest pain caused by muscle's demand for O2 & inability of arteries to supply it. - temporary situation --Stable or unstable · *Ischemia* is limited in duration does not cause damage · Happens w/ moderate - prolonged exertion · Stable plaque · Relieved by nitro or rest often managed with drug therapy

Chronic Stable Angina Pectoris

During a home visit, the nurse assesses a client who is taking hydrochlorothiazide and lisinopril for the treatment of hypertension. Which finding would indicate the nurse should inform the health care provider of a possible need to change medication therapy? • Blood pressure is 132/80 mm Hg. • Client has a persistent cough. • Potassium level is 4.1 mEq/L. • Client is experiencing nocturia.

Client has a persistent cough. A persistent cough is a side effect of the ACE inhibitor that may warrant a change to another antihypertensive medication. BP and potassium are within normal limits. The nurse assesses when the drug is taken and changes to an earlier time of administration.

The nurse is assessing a client with an atrial septal defect (ASD). Which requires immediate nursing intervention? • Fixed split S2, which does not vary with respiration • Client not taking his angiotensin-converting enzyme inhibitor this morning • Client having an uneven smile and facial droop • Client having tachycardia at a rate of 100 beats/min

Client having an uneven smile and facial droop A fixed S2 split is the hallmark of ASD. The neurologic finding of a facial droop could indicate embolization and stroke; the nurse should notify the healthcare provider immediately. If the client has missed a medication, the nurse should measure the vital signs and administer the medication as soon as possible; however, symptoms of stroke are the priority. The nurse should further assess tachycardia to determine the underlying cause, such as pain or fever, before intervening.

· Malnutrition · Immunosuppression · Large, open wounds · Mucous membrane fissures in prolonged contact with blood or drainage-soaked packing · GI ischemia · exposure to invasive procedures · malignancy · older than 85 · infection with resistance microorganisms · receiving cancer chemotherapy · alcoholism · diabetes mellitus · Chronic kidney disease · transplantation recipient · hepatitis · HIV/ AIDs

Conditions predisposing to sepsis & septic shock

- Impulses begin in *SA node* in right atrium at top near superior vena cava - Then move through atrium causing atrial depolarization (*P WAVE*) - Impulses then continue into *AV node* where it meets T-cells that slow impulse down before it gets to ventricles (*PR SEGMENT*) - Delay allows atria to contract & ventricles to fill ("*atrial kick*") - Impulse then moves into *Bundle of His* & branches all the way to *Purkinje fibers* - Fibers then cause ventricles to contract (*QRS*) - Ventricles then start to relax (*ST SEGMENT*) - And then repolarize (*T WAVE*) - It then starts all over again- Review 12 lead ECG & telemetry in your book - look at lead placement SA node --contract (S1) AV-pause Purkinje fibers- ventricles contract (S2)

Conduction

Categorized by blood flow pattern *Disorders w/ decreased pulmonary blood flow* Tetralogy of Fallot Tricuspid atresia *Disorders with increased pulmonary blood flow* Patent ductus arteriosus Atrial septal defect Ventricular septal defect *Obstructive disorders* Coarctation of the aorta Aortic stenosis Pulmonary stenosis *Mixed disorders* Transposition of the great vessels Total anomalous pulmonary venous return Truncus arteriosus Hypoplastic left heart syndrome

Congenital Heart Disease Categories

Echo, cardiac cath, chest xray

Congenital Heart Disease Labs/Diagnostics

Depends on the severity Surgical correction Prostaglandin infusion will maintain patency of the ductus arteriosus, improving pulmonary blood flow Tube feedings for nutrition

Congenital Heart Disease Management

Abnormal development Opening, connections, or narrowed areas May result in HF Chromosomal alterations Maternal exposure Toxins, infections, chronic illnesses, alcohol

Congenital heart disease

- ability to contract - Muscle fibers shorten on impulse causing contraction

Contractility

ability to contract - Muscle fibers shorten on impulse causing contraction

Contractility -

A nurse is caring for a client with type 2 diabetes who has had a myocardial infarction (MI) and is reporting nausea, vomiting, dyspnea, and substernal chest pain. Which of the following is the priority intervention? • Reduce the nausea and vomiting and stabilize the blood glucose. • Control the pain and support breathing and oxygenation. • Decrease the anxiety and reduce the workload on the heart. • Monitor and manage potential complications.

Control the pain and support breathing and oxygenation. Support of breathing and ensuring adequate oxygenation are the two most important priorities. Reducing the substernal pain is also important because upset and anxiety will increase the demand for oxygen in the body. Controlling nausea, vomiting, and anxiety are all secondary in importance. Prevention of complications is important following initial stabilization and control of pain.

• Instructions w/ info on diet, activity, meds, wt monitoring & plans for worsening sx • Eval of L ventricular function • ACE or ARB • Smoking counseling

Core Measures from Joint Commission before D/C

· Single largest killer of American men & women · Includes chronic stable angina & acute coronary syndromes · Affects arteries that provide blood, O2 & nutrients to heart · Blood flow becomes blocked · Ischemia - insufficient O2 · Infarction - ischemia prolonged = cell death

Coronary Artery Disease

Which of the following client statements should the nurse evaluate as indicating the client's correct understanding of the causes of coronary artery disease (CAD)? "The leading cause of CAD is atherosclerosis." "There are many causes of CAD." "Cigarette smoking is the most common cause of CAD." "You will need to ask your healthcare provider about the causes of CAD."

Correct response: "The leading cause of CAD is atherosclerosis." Explanation: Atherosclerosis (plaque formation) is the leading cause of CAD. Cigarette smoking is the leading cause of lung cancer. Telling the client to ask the healthcare provider is not appropriate.

A client in the emergency department complains of squeezing substernal pain that radiates to the left shoulder and jaw. He also complains of nausea, diaphoresis, and shortness of breath. What is the nurse's priority action? Administer oxygen, attach a cardiac monitor, take vital signs, and administer sublingual nitroglycerin. Complete the client's registration information, perform an electrocardiogram, gain I.V. access, and take vital signs. Alert the cardiac catheterization team, administer oxygen, attach a cardiac monitor, and notify the physician. Gain I.V. access, give sublingual nitroglycerin, and alert the cardiac catheterization team.

Correct response: Administer oxygen, attach a cardiac monitor, take vital signs, and administer sublingual nitroglycerin. Explanation: Cardiac chest pain is caused by myocardial ischemia. Therefore the nurse should administer supplemental oxygen to increase the myocardial oxygen supply, attach a cardiac monitor to help detect life-threatening arrhythmias, and take vital signs to ensure that the client isn't hypotensive before giving sublingual nitroglycerin for chest pain. Registration information may be delayed until the client is stabilized. Alerting the cardiac catheterization team or the physician before completing the initial assessment is premature.

client is returning from the operating room after inguinal hernia repair. The nurse notes that he has fluid volume excess from the operation and is at risk for left-sided heart failure. Which sign or symptom indicates left-sided heart failure? Bibasilar crackles Jugular vein distention Right upper quadrant pain Dependent edema

Correct response: Bibasilar crackles Explanation: Bibasilar crackles are a sign of alveolar fluid, a sequelae of left ventricular fluid, or pressure overload and indicate left-sided heart failure. Jugular vein distention, right upper quadrant pain (hepatomegaly), and dependent edema are caused by right-sided heart failure, usually a chronic condition.

A nurse is caring for a client with a history of cardiac disease and type 2 diabetes. The nurse is closely monitoring the client's blood glucose level. Which medication is the client most likely taking? Procainamide Diltiazem hydrochloride Carvedilol Amiodarone

Correct response: Carvedilol Explanation: The nurse must monitor blood glucose levels closely in clients with type 2 diabetes who are taking beta-adrenergic blockers such as carvedilol, because beta-adrenergic blockers may mask the signs of hypoglycemia. The nurse should monitor QRS duration in clients taking procainamide and pulmonary function in clients taking amiodarone (because the drug may cause pulmonary fibrosis). Diltiazem may cause an increased PR interval or bradycardia.

A nurse notes that the client's PR interval is .17 and the QRS complex is .10. What action should the nurse take next? Document the findings. Administer the ordered nitroglycerin paste. Give 2 liters of oxygen via nasal cannula. Request a 12-lead electrocardiogram.

Correct response: Document the findings. Explanation: These are normal findings. The nurse should document the findings. A 12-lead ECG would be ordered if the client needs further evaluation in the event of an abnormal finding. Administering nitroglycerin is a routine intervention and not related to the measured PR and QRS intervals. Oxygen administration is not indicated in the presence of normal findings.

When a client has a troponin level of 0.9 ng/mL, which of the following nursing interventions should be implemented? Document the finding as the only action Encourage the client to ambulate Notify the healthcare provider Apply oxygen at 2 L/minute per nasal cannula

Correct response: Notify the healthcare provider Explanation: Troponin is a myocardial cell protein that is elevated in the serum when myocardial damage has occurred during a myocardial infarction. The healthcare provider should be immediately notified when the troponin level is > 0.1 ng/mL. The client should not be ambulated at this time. Applying oxygen is appropriate, although the use of a nasal cannula is not recommended.

`A physician orders digoxin for a client with heart failure. During digoxin therapy, which laboratory value may predispose the client to digoxin toxicity? Calcium level of 7.5 mg/dl (0.4 mmol/L) Magnesium level of 2.5 mg/dl (0.1 mmol/L) Sodium level of 152 mEq/L (152 mmol/L) Potassium level of 3.1 mEq/L (3.1 mmol/L)

Correct response: Potassium level of 3.1 mEq/L (3.1 mmol/L) Explanation: Conditions that may predispose a client to digoxin toxicity include hypokalemia (evidenced by a potassium level less than 3.5 mEq/L), hypomagnesemia (evidenced by a magnesium level less than 1.5 mEq/L), hypothyroidism, hypoxemia, advanced myocardial disease, active myocardial ischemia, and altered autonomic tone. Hypermagnesemia (evidenced by a magnesium level greater than 2.5 mEq/L), hypercalcemia (evidenced by an ionized calcium level greater than 5.3 mg/dl), and hypernatremia (evidenced by a sodium level greater than 145 mEq/L) aren't associated with a risk of digoxin toxicity.

A client has an International normalized ratio (INR) of 1.6, creatine kinase-MB (CK-MB) of 90 μ/L, troponin 2.1 ng/L, and myoglobin 90 μg/L. Which of the following results requires the nurse to take action? Myoglobin 90 μg/L CK-MB of 90 μ/L INR of 1.6 Troponin of 2.1 ng/L

Correct response: Troponin of 2.1 ng/L Explanation: Troponins I and T are cardiac enzymes that are only released when the cardiac muscle is damaged. Elevation of these values above the respective reference ranges of 0-0.1 ng/L or 0-0.2 ng/L indicates a myocardial infarction. Myoglobin is released when muscle cells are damaged. Myoglobin may rise above the normal level of 0-90 μg/L with a myocardial infarction (MI) but is not a clear indicator of MI because it can also rise during strenuous exercise, traumatic injury, and intramuscular injections. CK-MB will rise following MI, but may be elevated by events that also raise myoglobin. A normal range for CK-MB is between 30 and 170 μ/L. The INR test is a measure of blood clotting. An INR value of 1.6 is within the normal range.

The nurse is assessing a client who had an abdominal aortic aneurysm repair 2 hours ago. Which finding warrants further evaluation? an arterial blood pressure of 80/50 mm Hg a BUN of 26 mg/dl (26 mmol/L) and creatinine of 1.2mg/dl (1.2 ?mol/L) absent bowel sounds and mild abdominal distension +1 pedal pulses in bilateral lower extremities

Correct response: an arterial blood pressure of 80/50 mm Hg Explanation: A blood pressure of 80/50 mm Hg in a client who has just had surgical repair of an abdominal aortic aneurysm warrants further evaluation as this indicates decreased perfusion to the brain, heart, and kidneys. A BUN of 26 and a creatinine of 1.2 are normal findings. While +1 pedal pulses may be an abnormal finding, it is not uncommon, and it is important to compare this finding to previous assessments and note if this is a change of the strength of the pedal pulses. Absent bowel sound and mild abdominal distension is expected for a client immediately following surgery. However this finding should be monitored as it could indicate a paralytic ileus.

A client arrives in the emergency department with an ischemic stroke. Because the health care team is considering administering tissue plasminogen activator (t-PA) administration, the nurse should first: identify the time of onset of the stroke. determine if the client is scheduled for any surgical procedures. ask what medications the client is taking. complete a history and health assessment.

Correct response: identify the time of onset of the stroke. Explanation: Studies show that clients who receive recombinant t-PA treatment within 3 hours after the onset of a stroke have better outcomes. The time from the onset of a stroke to t-PA treatment is critical. A complete health assessment and history is not possible when a client is receiving emergency care. Upcoming surgical procedures may need to be delayed because of the administration of t-PA, which is a priority in the immediate treatment of the current stroke. While the nurse should identify which medications the client is taking, it is more important to know the time of the onset of the stroke to determine the course of action for administering t-PA.

During physical assessment, the nurse should further assess the client for signs of atrial fibrillation when palpation of the radial pulse reveals: a weak, thready pulse two regular beats followed by one irregular beat irregular rhythm with pulse rate greater than 100 bpm. pulse rate below 60 bpm

Correct response: irregular rhythm with pulse rate greater than 100 bpm. Explanation: Characteristics of atrial fibrillation include pulse rate greater than 100 bpm, totally irregular rhythm, and no definite P waves on the ECG. During assessment, the nurse is likely to note the irregular rate and should report it to the health care provider (HCP) . A weak, thready pulse is characteristic of a client in shock. Two regular beats followed by an irregular beat may indicate a premature ventricular contraction.

An obese diabetic client has bilateral leg aching is to start a cardiac rehabilitation with an exercise program. Using which exercise equipment will be most helpful to the client? stationary bicycle stair climber treadmill elliptical trainer

Correct response: stationary bicycle Explanation: The stationary bicycle is the most appropriate training modality because it is a non-weight-bearing exercise. The time that the individual exercises on the stationary bicycle is increased with improved functional capacity. The other exercise equipment requires exercising while standing.

When assessing a client for early septic shock, the nurse should assess the client for which finding? cool, clammy skin warm, flushed skin increased blood pressure hemorrhage

Correct response: warm, flushed skin Explanation: Warm, flushed skin from a high cardiac output with vasodilation occurs in warm shock or the hyperdynamic phase (first phase) of septic shock. Other signs and symptoms of early septic shock include fever with restlessness and confusion; normal or decreased blood pressure with tachypnea and tachycardia; increased or normal urine output; and nausea and vomiting or diarrhea. Cool, clammy skin occurs in the hypodynamic or cold phase (later phase). Hemorrhage is not a factor in septic shock.

The nurse teaches the client with a demand pacemaker that the device functions by providing stimuli to the heart muscle: when the heart begins to beat irregularly. when the heart rate falls below a specified level. whenever ventricular fibrillation occurs. constantly, resulting in a predetermined heart rate.

Correct response: when the heart rate falls below a specified level. Explanation: A demand pacemaker functions only when the heart rate falls below a certain level. A fixed-rate pacemaker stimulates heart contractions at a constant rate independent of the client's heart rate. Fixed-rate pacemakers are much less common than demand pacemakers. A pacemaker is not used to provide cardioversion or defibrillation for cardiac arrhythmias (i.e., when the heart begins to beat irregularly). For a client with ventricular fibrillation, a potentially life-threatening arrhythmia, an implanted defibrillator is used.

The nurse has been instructing the client about how to prepare meals that are low in fat. Which of these comments would indicate the client needs additional teaching? "I'll eat water-packed tuna." "I will use a nonstick-coated pan when cooking." "I will eat more liver with onions." "I will avoid using steak sauce and catsup."

Correct response: "I will eat more liver with onions." Explanation: Liver and organ meats are high in cholesterol and saturated fat and should be limited. Water-packed tuna is one of the leanest fish available. Using a nonstick pan when cooking reduces the need for shortening or oil. Steak sauce and catsup are high in sodium and would likely accompany meals that include beef and other higher-fat meats.

A staff nurse is caring for a client who is a potential heart donor. The client's family is concerned that the recipient will have access to personal donor information. Which response by the nurse demonstrates knowledge of the organ donation process? "I will have the transplant coordinator speak with you to answer your questions." "There is never contact between the donor's family and the recipient." "The recipient is allowed to ask questions about the donor and have them answered." "It is important that the recipient know where to send Thank-You cards."

Correct response: "I will have the transplant coordinator speak with you to answer your questions." Explanation: The transplant coordinator, a specially trained person with knowledge of the donation, procurement, and transplantation process, typically speaks to family members about organ donation and answers their questions. Contact is permitted after the procedure with consent from the donor's family and the recipient. Typically, the transplant organization coordinates the communication. Confidentiality of the potential donor is always maintained unless the recipient and donor families both sign confidentiality waivers.

A newly admitted client states that he takes digoxin and warfarin. Which of the following would the nurse include in the discharge instructions? "Limit foods high in potassium, such as bananas." "Notify your healthcare provider if you experiences visual changes." "Report your morning and afternoon heart rates to your healthcare provider." "Increase your calorie intake if your appetite decreases."

Correct response: "Notify your healthcare provider if you experiences visual changes." Explanation: Hypokalemia can exacerbate digoxin toxicity so potassium should not be limited. The client will be taught the signs and symptoms of digoxin toxicity and what needs to be reported to the healthcare provider. Visual changes and anorexia are signs of digoxin toxicity and should be reported. The heart rate will not need to be verified twice a day. Anorexia is a symptom of digoxin toxicity so if the client is anorexic that should be reported to the healthcare provider.

A client has a heart rate of 170 beats/minute. The physician diagnoses ventricular tachycardia and orders lidocaine hydrochloride, an initial I.V. bolus of 50 mg followed in 5 minutes by a second 50-mg bolus, then continuous I.V. infusion at 2 mg/minute. The nurse can expect the client to begin experiencing an antiarrhythmic effect within: 1 to 2 minutes after I.V. bolus administration. 1 to 2 minutes after continuous I.V. infusion. 10 to 15 minutes after I.V. bolus administration. 10 to 15 minutes after continuous I.V. infusion.

Correct response: 1 to 2 minutes after I.V. bolus administration. Explanation: Lidocaine exerts its antiarrhythmic effect in 1 to 2 minutes after I.V. bolus administration. A continuous I.V. infusion will maintain lidocaine's antiarrhythmic effect for as long as the drip is used. Lidocaine provides antiarrhythmic effects for only 15 minutes after the I.V. infusion is stopped.

The nurse is providing discharge instructions to the client with peripheral vascular disease. The nurse should include which information in the discussion with this client? Select all that apply. Avoid prolonged standing and sitting. Limit walking so as not to activate the "muscle pump." Keep extremities elevated on pillows. Keep the legs in a dependent position. Use a heating pad to promote vasodilation.

Correct response: Avoid prolonged standing and sitting. Keep extremities elevated on pillows. Explanation: Elevating the extremities counteracts the forces of gravity and promotes venous return and reduces venous stasis. Walking is encouraged to activate the muscle pump and promote collateral circulation. Prolonged sitting and standing lead to venous stasis and should be avoided. Although heat promotes vasodilation, use of a heating pad is to be avoided to reduce the risk of thermal injury secondary to diminished sensation.

Which assessment findings would the nurse expect to find in the postoperative client experiencing fat embolism syndrome? Column A Column B Column C Column D

Correct response: Column B Explanation: Fat embolism syndrome is characterized by fever, tachycardia, tachypnea, and hypoxia. Arterial blood gas findings include a partial pressure of oxygen (PaO2) less than 60 mm Hg, with early respiratory alkalosis and later respiratory acidosis.

A client who suffered blunt chest trauma in a motor vehicle accident complains of chest pain, which is exacerbated by deep inspiration. On auscultation, the nurse detects a pericardial friction rub — a classic sign of acute pericarditis. The physician confirms acute pericarditis and begins appropriate medical intervention. To relieve chest pain associated with pericarditis, which position should the nurse encourage the client to assume? Semi-Fowler's Leaning forward while sitting Supine Prone

Correct response: Leaning forward while sitting Explanation: The nurse should encourage the client to lean forward, because this position causes the heart to pull away from the diaphragmatic pleurae of the lungs, helping relieve chest pain caused by pericarditis. The semi-Fowler's, supine, and prone positions don't cause this pulling-away action and therefore don't relieve chest pain associated with pericarditis.

A nurse just received a shift report for a group of clients on the telemetry unit. Which client should the nurse assess first? The client with a history of cardioversion for sustained ventricular tachycardia 2 days ago The client admitted with first-degree atrioventricular (AV) block whose cardiac monitor now reveals type II second-degree AV block The client with a history of heart failure who has bibasilar crackles and pitting edema in both feet The client admitted for unstable angina who underwent percutaneous coronary intervention (PCI) with stenting yesterday

Correct response: The client admitted with first-degree atrioventricular (AV) block whose cardiac monitor now reveals type II second-degree AV block Explanation: The client whose cardiac rhythm now shows type II second-degree AV block should be assessed first. The client's rhythm has deteriorated from first-degree heart block to type II second-degree AV block and may continue to deteriorate into a lethal form of AV block (known as complete heart block). The client who underwent cardioversion 2 days ago has likely had the underlying reason for the sustained ventricular tachycardia corrected. The client with a history of heart failure may have chronic bibasilar crackles and pitting edema of both feet. Therefore, assessing this client first is not necessary. The client who underwent PCI with stenting was at risk for reperfusion arrhythmias and/or bleeding from the arterial puncture site but could be considered to be stable 24 hours post procedure.

A 60-year-old comes into the emergency department with crushing substernal chest pain that radiates to the shoulder and left arm. The admitting diagnosis is acute myocardial infarction (MI). Admission prescriptions include oxygen by nasal cannula at 4 L/min, complete blood count (CBC), a chest radiograph, a 12-lead electrocardiogram (ECG), and 2 mg of morphine sulfate given IV. The nurse should first: administer the morphine. obtain a 12-lead ECG. obtain the blood work. obtain the chest radiograph.

Correct response: administer the morphine. Explanation: Although obtaining the ECG, chest radiograph, and blood work are all important, the nurse's priority action should be to relieve the crushing chest pain. Therefore, administering morphine sulfate is the priority action.

When assessing an individual with peripheral vascular disease, which clinical manifestation would indicate complete arterial obstruction in the lower left leg? aching pain in the left calf burning pain in the left calf numbness and tingling in the left leg coldness of the left foot and ankle

Correct response: coldness of the left foot and ankle Explanation: Coldness in the left foot and ankle is consistent with complete arterial obstruction. Other expected findings would include paralysis and pallor. Aching pain, a burning sensation, or numbness and tingling are earlier signs of tissue hypoxia and ischemia and are commonly associated with incomplete obstruction.

A nurse is educating a client who is at risk for coronary artery disease (CAD). The nurse knows that the client needs more education when he states that the risk factors that can be controlled or modified include: gender, family history, and older age. inactivity, stress, gender, and smoking. obesity, inactivity, diet, and smoking. stress, family history, and obesity.

Correct response: gender, family history, and older age. Explanation: The risk factors for coronary artery disease that can be controlled or modified include obesity, inactivity, diet, stress, and smoking. Gender, family history, and older age are risk factors that cannot be controlled.

Before discharge from the hospital after a myocardial infarction, a client is taught to exercise by gradually increasing the distance walked. Which vital sign should the nurse teach the client to monitor to determine whether to increase or decrease the exercise level? pulse rate blood pressure body temperature respiratory rate

Correct response: pulse rate Explanation: The client who is on a progressive exercise program at home after a myocardial infarction should be taught to monitor the pulse rate. The pulse rate can be expected to increase with exercise, but exercise should not be increased if the pulse rate increases more than about 25 bpm from baseline or exceeds 100 to 125 bpm. The client should also be taught to discontinue exercise if chest pain occurs.

While the nurse is assisting a client to ambulate as part of a cardiac rehabilitation program, the client has midsternal burning. The nurse should: stop and assess the client further. measure the client's blood pressure and heart rate. call for help and place the client in a wheelchair. administer nitroglycerin.

Correct response: stop and assess the client further. Explanation: The nurse should stop and assess the client further. A chair should be available for the client to sit down. Obtaining the client's blood pressure and heart rate are important when exercising. These values can be used to predict when the oxygen demand becomes greater than the oxygen supply. Calling for help is not necessary for the midsternal burning. If the health care provider (HCP) has prescribed nitroglycerin, the nurse can administer it; however, stopping the activity may restore the oxygen balance.

The nurse is caring for a patient suspected to have pulmonary edema. The nurse notes bilateral crackles, orthopnea, edema, and shortness of breath. After notifying the physician, the nurse should avoid which of the following while waiting for the physician to arrive? Elevate the head of bed to high-Fowler's position. Prepare to administer a diuretic. Provide supplemental oxygen if needed. Elevate the patient's legs to improve venous return.

Elevate the patient's legs to improve venous return. • This patient is showing signs of pulmonary edema caused by left-sided heart failure. • Elevating the patient's legs would increase venous return. This could overload the heart and worsen the patient's condition. • This question asked you to identify the action the nurse should avoid, so the following are incorrect options: • Incorrect: High-Fowler's position would improve the patient's ventilation. • Incorrect: Supplemental oxygen would help improve the patient's oxygenation. • Incorrect: Diuretics would help reduce pulmonary edema and fluid overload.

- respond to impulse Non pacemaker cells respond to pacemaker cells and depolarize

Excitability

Which signs and symptoms accompany a diagnosis of pericarditis? • Fever, chest discomfort, and elevated erythrocyte sedimentation rate (ESR) Low urine output secondary to left ventricular dysfunction • Lethargy, anorexia, and heart failure • Pitting edema, chest discomfort, and nonspecific ST-segment elevation

Fever, chest discomfort, and elevated erythrocyte sedimentation rate (ESR) Low urine output secondary to left ventricular dysfunction The classic signs and symptoms of pericarditis include fever, positional chest discomfort, nonspecific ST-segment elevation, elevated ESR, and pericardial friction rub. Low urine output secondary to left ventricular dysfunction lethargy, anorexia, heart failure and pitting edema, result from acute renal failure.

Pathology same as adults Tx same as adults *Assessment* Failure to gain wt or rapid wt gain Difficulty feeding, sucking, then tiring quickly ↓ # of wet diapers SOB Nasal flaring Other symptoms same as adult

Heart Failure

• Activates w/ ↓ blood flow to kidneys • vasoconstriction

Heart Failure Compensation by the Body *Renin-angiotensin system activation*

• ↑ catecholamines • Most immediate compensatory mechanism • ↑ HR & BP (vasoconstriction) o ↑ HR = ↑ CO for a short time o ↑ HR = ↑ O2 need of myocardium ○ Arteriosclerosis? HF worsen o BP ↑ w/ vasoconstriction but afterload makes heart work harder

Heart Failure Compensation by the Body *Sympathetic Nervous system stimulation*

A physician orders several drugs for a client with hemorrhagic stroke. Which drug order should the nurse question? • Methyldopa • Dexamethasone • Heparin sodium • Phenytoin

Heparin sodium Administering heparin, an anticoagulant, could increase the bleeding associated with hemorrhagic stroke. Therefore, the nurse should question this order to prevent additional hemorrhage in the brain. In a client with hemorrhagic stroke, the physician may use dexamethasone to decrease cerebral edema and pressure; methyldopa, to reduce blood pressure; and phenytoin, to prevent seizures.

• Determine *HR* (#beats in 6 sec x 10) • Determine *rhythm* • Find *P wave* --Is it present? --Is it consistent? --One for every QRS? --Smooth, round, upright? • Measure *PR interval* --Greater than 0.20 seconds? --Less than 0.12 seconds? --Constant? • Measure *QRS* --Greater than 0.12 seconds? --Similar in appearance? • *ST segment* --Is there elevation? • *T wave* --Shape, height, inversion? • *QT interval*

How to analyze ECG Strip

High dose ASA Single dose of IVIG Corticosteroid

Kawasaki Disease management

• Can be *systolic* or *diastolic* *Causes* • HTN • Coronary Artery Disease • Valvular Disease • Poor cardiac output Pulmonary congestion

Left Sided Heart Failure CHF

• *Left ventricle* cannot relax adequately "*stiffens*" • EF > 40% • Ventricle becomes ↓ able to work over time • 20-40% of all heart failure • Mainly seen in older adults & women w/ HTN & CAD *Same s/s of systolic*

Left Sided Heart Failure (CHF) *Diastolic*

• Heart can't contract forcefully enough to eject adequate amts of blood • *EF < 40%* • Tissue perfusion diminishes • Blood accumulates in pulmonary vessels *High risk for MI*

Left Sided Heart Failure (CHF) *Systolic*

• *Pulmonary* congestion • ↓ CO • Fatigue/weakness with activity • Dyspnea/orthopnea • Palpitations • *Cough* • S3 gallop • Crackles/ wheezes • Like pneumonia d/t fluid

Left Sided Heart Failure (CHF) *Systolic: Symptoms*

• Results from *rheumatic carditis* (rheumatic fever) & congenital abnormalities • Valve becomes stiff, chorade tendineae contract & shorten, valve opening *narrows* • L atrial pressure ↑ = atrium enlarges, *R sided HF*

Mitral Stenosis

• Dyspnea • Orthopnea • Palpitations • Dry cough • Hemoptysis • Pulmonary edema • Afib • Apical diastolic murmur

Mitral Stenosis *Signs & Symptoms*

• Valvular leaflets enlarge and prolapse into the L atrium • Usually benign may progress to mitral regurgitation • Causes - congenital, familial

Mitral Valve Prolapse

• ? chest pain, palpitations • Click & murmur heard at apex

Mitral Valve Prolapse *Signs & Symptoms*

· The most serious · Undiagnosed or untreated angina can lead to MI · Occurs when myocardial tissue abruptly & severely deprived of O2 · Blood flow reduced by 80-90% resulting in ischemia *Non - ST segment elevation MI (NSTEMI)* · Have ST & T wave changes - indication of myocardial necrosis · Elevated cardiac enzymes · Causes - coronary vasospasm, spontaneous dissection, sluggish blood flow *ST-elevation MI (STEMI)* · ST elevation on ECG · Attributed to plaque thrombus - 100% blocked

Myocardial Infarction

· Does not occur instantly - evolves over hours · Hypoxemia from the ischemia leads to vasodilation of blood vessels · Catecholamines release in response to hypoxia ↑ HR, contractility and afterload --These ↑ O2 requirements - heart already O2 deprived --Lead to ventricular dysrhythmias --Area of infarction spreads *Ventricular remodeling* · Changes to the heart · 6 hrs after infarct - heart appears blue and swollen · 48 hrs - infarcted area turns grey with yellow streaks (neutrophils) · 8-10 days granulation tissue forms at edges of necrotic tissue · 2-3 months - necrotic area shrinks forms thin firm scar · Changes the size and shape of the heart *Obstruction of the L anterior descending artery* · Anterior or septal MI · Highest mortality rate · Most likely to have L ventricular failure & dysrhythmias *Obstruction of the circumflex artery* · Posterior wall or lateral MI · Sinus dysrhythmias *Obstruction of R coronary artery* · Inferior wall MI · Significant damage to R ventricle

Myocardial Infarction What Happens

Regular rhythm rate 60-100 QRS duration normal P wave preceding each QRS PR interval normal

Normal Sinus Rhythm

The nurse is reviewing the electrocardiogram of a client who has elevated ST segments visible in leads II, III, and aVf. Which is the nurse's best action? • Document the finding in the medical record • Determine whether the rhythm is irregular, coinciding with inspiration and expiration • Teach the client about risks for coronary artery disease • Notify the healthcare provider

Notify the healthcare provider Leads II, III, and aVF record electrical events on the inferior surface of the left ventricle; elevated ST sements indicate that the client is experiencing a myocardial infarction. The healthcare provider should be notified. Teaching should be delayed until the client is stable. An irregular heart rhythm that varies with respiration—sinus arrhythmia—is a normal variation of sinus rhythm; there is no intervention needed.

• Temporary pacing - demand pacing • Nonsurgical • Provides a timed electrical stimulus • Transcutaneous pacing - fixed rate pacing • Two large external electrodes attached to an external pulse generator • Only fires when the HR goes lower than set on the machine • Emergency measure *Permanent pacemaker* • Powered by a lithium battery • Average life span of 10 years • Combination pacemaker/defibrillator devices available •Placed in a pocket creased in the right or left subclavicular area

Pacemakers

A client who is being discharged after a hospitalization for thrombophlebitis will be riding home in a car. During the 2-hour care ride, what should the nurse should advise the client to do? • Perform arm circles while riding in the car. • Perform ankle pumps and foot range-of-motion exercises. • Elevate her legs while riding in the car. • Take an ambulance home.

Perform ankle pumps and foot range-of-motion exercises. Performing active ankle and foot range-of-motion exercises periodically during the ride home will promote muscular contraction and provide support to the venous system. It is the muscular action that facilitates return of the blood from the lower extremities, especially when in the dependent position. Arm circle exercises will not promote circulation in the leg. It is not necessary for the client to elevate the legs as long as the client does not occlude blood flow to the legs and does the leg exercises. It is not necessary to take an ambulance because the client is able to sit in the car safely.

*Acute* • Inflammation of the pericardium • Comes from infective organisms or MI *Chronic* • A fibrous thickening of the pericardium *Causes* • TB, radiation, trauma, renal failure, metastatic cancer • Pericardium becomes rigid, Ventricles cannot fill, results in HF

Pericarditis

A physician orders digoxin for a client with heart failure. During digoxin therapy, which laboratory value may predispose the client to digoxin toxicity? • Magnesium level of 2.5 mg/dl (0.1 mmol/L) • Calcium level of 7.5 mg/dl (0.4 mmol/L) • Sodium level of 152 mEq/L (152 mmol/L) • Potassium level of 3.1 mEq/L (3.1 mmol/L)

Potassium level of 3.1 mEq/L (3.1 mmol/L) Conditions that may predispose a client to digoxin toxicity include hypokalemia (evidenced by a potassium level less than 3.5 mEq/L), hypomagnesemia (evidenced by a magnesium level less than 1.5 mEq/L), hypothyroidism, hypoxemia, advanced myocardial disease, active myocardial ischemia, and altered autonomic tone. Hypermagnesemia (evidenced by a magnesium level greater than 2.5 mEq/L), hypercalcemia (evidenced by an ionized calcium level greater than 5.3 mg/dl), and hypernatremia (evidenced by a sodium level greater than 145 mEq/L) aren't associated with a risk of digoxin toxicity.

Ventricle beating before it should uniform- look exactly the same -increase w/age significant if other problems occur -treat underlying problem (us F/E imb) occur repetitively PVS do not count towards HeartBeart

Premature Ventricular Contractions (PVC) Uniform/unifocal

multiform- look different

Premature Ventricular Contractions (PVC) Multiform/multifocal

A patient with a recent myocardial infarction (MI) has asked if he can talk to the nurse about sex. What should the nurse do? Instruct staff to avoid the patient's room. Report the incident to the police. Report the incident to the nurse manager. Prepare patient teaching information.

Prepare patient teaching information. • It is expected for patients to be curious about when it may be safe for them to resume normal sexual activity. • The nurse should prepare patient teaching information to give to the patient and instruct him that he may resume normal sexual activity based on his symptoms. This may be as soon as two weeks, but the physician will want him to wait until he is feeling well and to avoid any vigorous activity (including sexual activity) if the patient is experiencing any shortness of breath, heart palpitations, chest pain, or lightheadedness. • The incident should not be interpreted as malicious or be reported without first assessing the situation. • Staff should never avoid a patient's room.

A physician orders blood coagulation tests to evaluate a client's blood-clotting ability. The nurse knows that such tests are important in assessing clients at risk for thrombi, such as those with a history of atrial fibrillation, infective endocarditis, prosthetic heart valves, or myocardial infarction. Which test determines a client's response to oral anticoagulant drugs? • Partial thromboplastin time (PTT) • Platelet count • Prothrombin time (PT) • Bleeding time

Prothrombin time (PT) PT determines a client's response to oral anticoagulant therapy. This test measures the time required for a fibrin clot to form in a citrated plasma sample following addition of calcium ions and tissue thromboplastin and compares this time with the fibrin-clotting time in a control sample. The physician should adjust anticoagulant dosages as needed, to maintain PT at 1.5 to 2.5 times the control value. Bleeding time indicates how long it takes for a small puncture wound to stop bleeding. The platelet count reflects the number of circulating platelets in venous or arterial blood. PTT determines the effectiveness of heparin therapy and helps physicians evaluate bleeding tendencies. Physicians diagnose appoximately 99% of bleeding disorders on the basis of PT and PTT values.

A client in the emergency department with a cardiac dysrhythmia is to receive a prescribed dose of procainamide. For which of the following conditions in the client's history would the nurse withhold the medication until the order is clarified? Select all that apply. Hypertension • Second-degree heart block • Myasthenia gravis • Systemic lupus erythematosus (SLE) • Pulmonary embolus

Second-degree heart block Myasthenia gravis Systemic lupus erythematosus (SLE) Procainamide is a cardiac antidysrhythmic medication that can prolong the QRS and QT intervals. The drug is contraindicated in complete heart block and SLE. It should be used cautiously in heart failure and myasthenia gravis, as those conditions may be worsened by the medication.

· ↑ RR · ↓ CO · Fever can be high or none depending on pt · ↑ WBC · Microthrombi form making some organs hypoxic · If treated and caught here, good outcome

Sepsis Symptoms

· Temp > 100.4 or <96.8 · HR > 90 · Resp > 20 or PaCO2 < 32mm Hg · Abnormal WBCs (>12,000/mm3 or <4000/mm3 or >10% segs or >10% bands) · Sepsis is considered to be present if 2 or more SIRS criteria are present along with any known infection & 1 or more of these clinical manifestations: · Hypotension · urine output < fluid intake · positive fluid balance · decreased capillary refill · hyperglycemia (>120 if not diabetic) · unexplained change in mental status · rising serum creatinine level in person w/o kidney problems (>2.0 (men) >1.4 (women))

Sepsis with Systemic Inflammatory Response Syndrome SIRS criteria

· Antibiotics · O2 · Fluids · Insulin · Drugs used to treat symptoms · Blood/ platelets

Septic Shock Treatment

· Identify Pts at risk · Age 65+ · Use aseptic technique with open skin · Remove catheters and IVs when no longer needed · Try to detect early for best outcome · Watch the patient, often in early stages behavior will change, pt will become *restless, fidgety* · Labs

Severe Sepsis Prevention

· ↑CO · ↑ RR · ↑ Systolic BP · Patient still has warm extremities · Normal WBC (used up) · ↓ O2 · ↓ to no urine output · With interventions septic shock can be prevented but it is going downhill quickly

Severe Sepsis Symptoms

*Monitoring* · ABG monitoring · Oxygenation · Hemodynamic monitoring · IV therapy · Blood · BP · HR/VS · Level of consciousness · Cardiac rhythm · Circulation checks · Symptoms of organ failure · IV access · Meds as ordered · High flow oxygen · Surgery to repair the cause if possible

Shock *Interventions*

• Moderate vasoconstriction • HR increase • Decreased pulse pressure • RAA activation • Mild hyperkalemia *Symptoms* • Thirst, anxiety, restlessness, tachycardia, ↑ RR, ↓ urine output, ↓Systolic BP, ↑ diastolic BP, cool extremities, decrease O2

Shock Stages *Nonprogressive*

(known as the pacemaker of the heart) - Sympathetic and parasympathetic nervous system increases and decreases rate of discharge of the SA node - changes HR

Sinoatrial node (SA node) - SA node

Rate 60-100 Irregular rhythm P wave before each QRS PR interval normal QRS duration normal · Results from changes in thoracic pressure during breathing; hr increases during inspiration and decreases during expiration · Considered normal rhythm

Sinus Arrhythmia

Regular rhythm Rate > 100 QRS duration normal P wave preceding each QRS PR interval normal - Can be normal or abnormal - look at pt - Initially increases CO and BP - Unable to fill ventricles completely, decreased perfusion and CO - Causes, anxiety, pain, stress, fever, anemia, hypoxemia, hyperthyroidism, drugs, low BP, hypovolemic shock - Look for hypovolemia, dehydration Vagal if needed

Sinus Tachycardia

A physician admits a client to the health care facility for treatment of an abdominal aortic aneurysm. When planning this client's care, which goal should the nurse keep in mind as she formulates interventions? • Increasing blood pressure and reducing mobility • Decreasing blood pressure and increasing mobility • Increasing blood pressure and monitoring fluid intake and output • Stabilizing heart rate and blood pressure and easing anxiety

Stabilizing heart rate and blood pressure and easing anxiety For a client with an aneurysm, nursing interventions focus on preventing aneurysm rupture by stabilizing heart rate and blood pressure. Easing anxiety also is important because anxiety and increased stimulation may raise the heart rate and boost blood pressure, precipitating aneurysm rupture. The client with an abdominal aortic aneurysm is typically hypertensive, so the nurse should take measures to lower blood pressure, such as administering antihypertensive agents, as ordered, to prevent aneurysm rupture. To sustain major organ perfusion, the client should maintain a mean arterial pressure of at least 60 mm Hg. Although the nurse must assess each client's mobility individually, most clients need bed rest when initially attempting to gain stability.

A nurse is caring for a client with advanced heart failure. He can't care for himself and hasn't been able to eat for the past week because of dyspnea. The client doesn't want a feeding tube inserted and expresses his desire for "nature to take its course." The client's family is pleading with him to have a feeding tube inserted. What is the most appropriate action for the nurse to take? • Schedule a conference to help the client and his family reach a consensus about the feeding tube. • Ask a priest to talk with the client about the importance of preserving life. • Talk with the client's family about the client's right to decide for himself. • Schedule feeding tube placement and hope that she can persuade the client to agree to it.

Talk with the client's family about the client's right to decide for himself. Advocating for a client's wishes is a key nursing role. It's especially important when a client's family disagrees with his wishes. The nurse should be sure that the client has all the information he needs to make an informed decision. Then she should support his decision. She shouldn't contact a clergyman without the client's consent, call a family conference, or schedule intubation in violation of the client's wishes.

A client in the intensive care unit (ICU) is on a dobutamine drip. During an assessment the client states, "I was feeling better but now my chest is tight and I feel like my heart is skipping." Physical assessment reveals a heart rate of 110 beats per minute and blood pressure of 160/98 mm Hg. What is the nurse's immediate concern for this client? • The dobutamine may need to be decreased. • The client is experiencing an allergic reaction to the dobutamine. • The client is experiencing an exacerbation of the heart failure. • The dosage of the dobutamine needs to be increased.

The dobutamine may need to be decreased. Dobutamine is a vasoactive adrenergic that works by increasing myocardial contractility and stroke volume in order to increase the cardiac output in heart failure clients. A serious side effect of adrenergic drugs is the worsening of a preexisting cardiac disorder. Increasing the dosage of the drug will worsen the problem. The client shows not symptoms of allergic reaction or heart failure.

The client with heart failure asks the nurse about the reason for taking enalapril maleate. The nurse should tell the client: • "This drug will constrict your blood vessels and keep your blood pressure from getting too low." • "This drug will slow your heart rate down." • "This drug helps your heart beat more forcefully." • "This drug will dilate your blood vessels and lower your blood pressure."

This drug will dilate your blood vessels and lower your blood pressure." Enalapril maleate is an angiotensin-converting enzyme inhibitor that prevents conversion of angiotensin I to angiotensin II. Angiotensin II is a potent vasoconstrictor and also contributes to aldosterone secretion. Thus, enalapril decreases blood pressure through systemic vasodilation. Enalapril does not cause increased vasoconstriction, which would raise blood pressure. The medication has no effect on myocardial contractility or the heart's conduction system.

• < 1 yr to live • < 65 • Class III or IV • < slightly ↑ pulmonary vascular resistance • No infection • Stable psychosocial • No drug/ alcohol abuse

Transplant Candidate selection

• Watch for *cardiac tamponade* --• Too much fluid - stops heart from beating • Bleeding • Orthostatic hypotension • Immunosuppressant's

Transplant Post op

· Chest pain or discomfort that occurs at rest or with exertion and causes severe activity limitation · Lasts longer than 15 min · Poorly relieved with rest or nitro · Have ST changes on EKG but no changes in troponin or · CK levels

Unstable Angina Pectoris

• Chaotic disorganized activity difficult to discern rate • Treat: defibrillation, antidysrhythmic drugs, CPR • emergency- no pulse • causes: MY, hypokalemia, hypomagnesemia, hemorrhage, Rapid, SVT

Ventricular Fibrillation (V-fib)

Regular rhythm rate usually 140-200 QRS prolonged P waves not seen Treat: defibrillation/cardioversion, antidysrhythmic drugs (lidocaine, amiodarone) • Seen: heart disease, MI, cardiomyopathy, hypovolemia, hypokalemia, arterial HTN, *hypomagnesemia*, drug toxicity, HF, cocaine • Wil ll deteriorate if doent convert • stable- has pulse • unstable- does not

Ventricular Tachycardia (VT, V-tach)

A client with aortic stenosis tells the nurse, "I have been feeling so tired lately that I take a nap in my recliner every afternoon." On assessment, the nurse notes apical heart sounds 2 cm left of the midclavicular line, crackles in lower lung fields during respiration, blood pressure 110/90 mm Hg, and weight gain of 2.5 kg (5.5 lb) in 24 hours. Which of the following assessments requires further action? • Apical heart sounds 2 cm to the left of midclavicular line • Weight gain of 2.5 kg (5.5 lb) in 24 hours • Blood pressure 110/90 mm Hg • Crackles in lower lung fields during inspiration

Weight gain of 2.5 kg (5.5 lb) in 24 hours Aortic stenosis leads to left ventricular enlargement and eventually to heart failure. Signs of heart failure include rapid weight gain, a shift of the apical pulse to the left of the midclavicular line, narrowed pulse pressure, and adventitious lung sounds. The nurse must intervene for rapid weight gain of more than 1 kg in 24 hours, which indicates fluid retention from worsening heart failure.

As an initial step in treating a client with angina, the health care provider (HCP) prescribes nitroglycerin tablets, 0.3 mg given sublingually. This drug's principal effects are produced by: • antispasmodic effects on the pericardium. • causing an increased myocardial oxygen demand. • vasodilation of peripheral vasculature. • improved conductivity in the myocardium.

`vasodilation of peripheral vasculature. Nitroglycerin produces peripheral vasodilation, which reduces myocardial oxygen consumption and demand. Vasodilation in coronary arteries and collateral vessels may also increase blood flow to the ischemic areas of the heart. Nitroglycerin decreases myocardial oxygen demand. Nitroglycerin does not have an effect on pericardial spasticity or conductivity in the myocardium.

While auscultating the heart sounds of a client with heart failure, the nurse hears an extra heart sound immediately after the second heart sound (S2). The nurse should document this as: • a murmur. • a fourth heart sound (S4). • a third heart sound (S3). • a first heart sound (S1). An S3 is heard following an S2, which commonly occurs in clients experiencing heart failure and results from increased filling pressures. An S1 is a normal heart sound made by the closing of the mitral and tricuspid valves. An S4 is heard before an S1 and is caused by resistance to ventricular filling. A murmur is heard when there is turbulent blood flow across the valves.

a third heart sound (S3). An S3 is heard following an S2, which commonly occurs in clients experiencing heart failure and results from increased filling pressures. An S1 is a normal heart sound made by the closing of the mitral and tricuspid valves. An S4 is heard before an S1 and is caused by resistance to ventricular filling. A murmur is heard when there is turbulent blood flow across the valves.

A client with left-sided heart failure complains of increasing shortness of breath and is agitated and coughing up pink-tinged, foamy sputum. The nurse should recognize these findings as signs and symptoms of: • pneumonia. • cardiogenic shock. • acute pulmonary edema. • right-sided heart failure.

acute pulmonary edema. Shortness of breath, agitation, and pink-tinged, foamy sputum signal acute pulmonary edema. This condition results when decreased contractility and increased fluid volume and pressure in clients with heart failure drive fluid from the pulmonary capillary beds into the alveoli. In right-sided heart failure, the client would exhibithepatomegaly, jugular vein distention, and peripheral edema. In pneumonia, the client would have a temperature spike and sputum that varies in color. Cardiogenic shock is indicated by signs of hypotension andtachycardia.

A client has a blockage in the proximal portion of a coronary artery. After learning about treatment options, the client decides to undergo percutaneous transluminal coronary angioplasty (PTCA). During this procedure, the nurse expects to administer an: • anticonvulsant. • antihypertensive. • antibiotic. • anticoagulant.

anticoagulant. During PTCA, the client receives heparin, an anticoagulant, as well as calcium agonists, nitrates, or both, to reduce coronary artery spasm. Nurses don't routinely give antibiotics during this procedure; however, because the procedure is invasive, the client may receive prophylactic antibiotics to reduce the risk of infection. An antihypertensive may cause hypotension, which should be avoided during the procedure. An anticonvulsant isn't indicated because this procedure doesn't increase the risk of seizures.

The nurse is caring for a client diagnosed with an anterior myocardial infarction 2 days ago. Upon assessment, the nurse identifies a systolic murmur at the apex. The nurse should first: • obtain a 12-lead electrocardiogram. • evaluate heart sounds with the client leaning forward. • draw an arterial blood gas. • assess for changes in vital signs.

assess for changes in vital signs. The nurse should first obtain vital signs as changes in the vital signs will reflect the severity of the sudden drop in cardiac output: decrease in blood pressure, increase in heart rate, and increase in respirations. Infarction of the papillary muscles is a potential complication of an MI causing ineffective closure of the mitral valve during systole. Mitral regurgitation results when the left ventricle contracts and blood flows backward into the left atrium, which is heard at the fifth intercostal space, left midclavicular line. The murmur worsens during expiration and in the supine or left-side position, and can best be heard when the client is in these positions, not with the client learning forward. A 12-lead ECG views the electrical activity of the heart; an echocardiogram views valve function.

An older adult with a history of heart failure is admitted to the emergency department with pulmonary edema. On admission, what should the nurse assess first? • blood pressure • skin breakdown • serum potassium • level urine output It is a priority to assess blood pressure first because people with pulmonary edema typically experience severe hypertension that requires early intervention. The client probably does not have skin breakdown, but when the client is stable and when the nurse obtains a complete health history, the nurse should inspect the client's skin for any signs of breakdown; however, when the client is stable, the nurse should inspect the skin. Potassium levels are not the first priority. The nurse should monitor urine output after the client is stable.

blood pressure It is a priority to assess blood pressure first because people with pulmonary edema typically experience severe hypertension that requires early intervention. The client probably does not have skin breakdown, but when the client is stable and when the nurse obtains a complete health history, the nurse should inspect the client's skin for any signs of breakdown; however, when the client is stable, the nurse should inspect the skin. Potassium levels are not the first priority. The nurse should monitor urine output after the client is stable.

Metoprolol is added to the pharmacologic therapy of a woman with diabetes diagnosed with stage 2 hypertension and initially treated with furosemide and ramipril. An expected therapeutic effect is: • decrease in heart rate. • lessening of fatigue. • improvement in blood sugar levels. • increase in urine output

decrease in heart rate. The effect of a beta blocker is a decrease in heart rate, contractility, and afterload, which leads to a decrease in blood pressure. The client at first may have an increase in fatigue when starting the beta blocker. The mechanism of action does not improve blood sugar or urine output.

A client has a history of heart failure and has been prescribed furosemide, digoxin, and potassium chloride. The client has nausea, blurred vision, headache, and weakness. The nurse notes that the client is confused. The telemetry strip shows first-degree atrioventricular block. The nurse should assess the client for signs of: • digoxin toxicity. • fluid deficit. • hyperkalemia. • pulmonary edema.

digoxin toxicity. Early symptoms of digoxin toxicity include anorexia, nausea, and vomiting. Visual disturbances can also occur, including double or blurred vision and visual halos. Hypokalemia is a common cause of digoxin toxicity associated with arrhythmias because low serum potassium can enhance ectopic pacemaker activity. Although vomiting can lead to fluid deficit, given the client's history, the vomiting is likely due to the adverse effects of digoxin toxicity. Pulmonary edema is manifested by dyspnea and coughing.

Following a myocardial infarction, a client develops an arrhythmia and requires a continuous infusion of lidocaine. To monitor the effectiveness of the intervention, the nurse should focus primarily on the client's: • electrocardiogram (ECG). • lidocaine level. • troponin level. • blood pressure.

electrocardiogram (ECG). Lidocaine is an antiarrhythmic and is given for the treatment of cardiac irritability and ventricular arrhythmias. The best indicator of its effectiveness is a reduction in or disappearance of ventricular arrhythmias as seen on an ECG. Lidocaine level will be monitored but it is not the primary focus; troponin level monitors myocardial damage. Blood pressure which can drop on lidocaine does need to be monitored but the focus should be the ECG to evaluate the effectiveness of the medication.

During physical assessment, the nurse should further assess the client for signs of atrial fibrillation when palpation of the radial pulse reveals: • two regular beats followed by one irregular beat • irregular rhythm with pulse rate greater than 100 bpm. • pulse rate below 60 bpm • a weak, thready pulse Characteristics of atrial fibrillation include pulse rate greater than 100 bpm, totally irregular rhythm, and no definite P waves on the ECG. During assessment, the nurse is likely to note the irregular rate and should report it to the health care provider (HCP) . A weak, thready pulse is characteristic of a client in shock. Two regular beats followed by an irregular beat may indicate a premature ventricular contraction.

irregular rhythm with pulse rate greater than 100 bpm. Characteristics of atrial fibrillation include pulse rate greater than 100 bpm, totally irregular rhythm, and no definite P waves on the ECG. During assessment, the nurse is likely to note the irregular rate and should report it to the health care provider (HCP) . A weak, thready pulse is characteristic of a client in shock. Two regular beats followed by an irregular beat may indicate a premature ventricular contraction.

The nurse should assess the client for digoxin toxicity if serum levels indicate that the client has a: • low sodium level. • high glucose level. • high calcium level. • low potassium level.

low potassium level. A low serum potassium level (hypokalemia) predisposes the client to digoxin toxicity. Because potassium inhibits cardiac excitability, a low serum potassium level would mean that the client would be prone to increased cardiac excitability. Sodium, glucose, and calcium levels do not affect digoxin or contribute to digoxin toxicity.

• no rate • Treat: pacemaker, compressions, Epinephrine • Do NOT shock- CPR --only for VF or pulseless VT • VTach run, then stops, atrial still firing, Still considered asystole Cause: myocardial Hypoxia, severe hyperkalemia CPR

no rate

A nurse is caring for a client with acute pulmonary edema. To immediately promote oxygenation and relieve dyspnea, the nurse should: • administer oxygen. • have the client take deep breaths and cough. • place the client in high Fowler's position. • perform chest physiotherapy.

place the client in high Fowler's position. The high Fowler's position will initially promote oxygenation in the client and relieve shortness of breath. Additional measures include administering oxygen to increase oxygen content in the blood. Deep breathing and coughing will improve oxygenation postoperatively but may not immediately relieve shortness of breath. Chest physiotherapy results in expectoration of secretions, which isn't the primary problem in pulmonary edema.

The nurse is assessing a client with superficial thrombophlebitis in the greater saphenous vein of the left leg. The client has "aching" in the leg. Which finding indicates the nurse should contact the health care provider (HCP) to request a prescription to improve the client's comfort? • brown discoloration of the skin with edema in the lower left leg • dark, protruding veins of both legs that are uncomfortable when standing • absence of pain or swelling when the client dorsiflexes the left foot • red, warm, palpable linear cord along the vein that is painful on palpation

red, warm, palpable linear cord along the vein that is painful on palpation Superficial thrombophlebitis is associated with pain, warmth, and erythema. The nurse can request a prescription for warm packs to relieve the pain. Venous insufficiency causes edema and a brown discoloration of the lower leg. Varicose veins are dark, protruding veins, and symptoms of discomfort increase with standing. Pain on dorsiflexion of the foot indicates deep vein thrombosis; the client does not indicate having this pain.

Alteplase recombinant, or tissue plasminogen activator (t-PA), a thrombolytic enzyme, is administered during the first 6 hours after onset of myocardial infarction (MI) to: • revascularize the blocked coronary artery. • reduce coronary artery vasospasm. • control the arrhythmias associated with MI. • control chest pain.

revascularize the blocked coronary artery. The thrombolytic agent t-PA, administered intravenously, lyses the clot blocking the coronary artery. The drug is most effective when administered within the first 6 hours after onset of MI. The drug does not reduce coronary artery vasospasm; nitrates are used to promote vasodilation. Arrhythmias are managed by antiarrhythmic drugs. Surgical approaches are used to open the coronary artery and re-establish a blood supply to the area.

A client with peripheral vascular disease has poor circulation. The nurse should assess the client for changes in: (Select all that apply.) • skin temperature • fluid intake • pain in extremity • nail bed color • nausea Maintaining circulation is critical in individuals with peripheral vascular disease. Skin and nail bed color and temperature will reveal the degree to which the extremity is receiving blood flow. Clients with peripheral vascular disease also usually have a certain amount of pain, especially when the oxygen demand becomes greater than oxygen supply, such as with walking or exercising. Fluid intake and nausea are unrelated to peripheral circulation.

skin temperature pain in extremity nail bed color Maintaining circulation is critical in individuals with peripheral vascular disease. Skin and nail bed color and temperature will reveal the degree to which the extremity is receiving blood flow. Clients with peripheral vascular disease also usually have a certain amount of pain, especially when the oxygen demand becomes greater than oxygen supply, such as with walking or exercising. Fluid intake and nausea are unrelated to peripheral circulation.

The nurse teaches a client with heart failure to take oral furosemide in the morning. The primary reason for this is to prevent: • nausea or vomiting. • electrolyte imbalances. • sleep disturbances during the night. • excretion of excessive fluids accumulated during the night. .

sleep disturbances during the night When diuretics are given early in the day, the client will void frequently during the daytime hours and will not need to void frequently during the night. Therefore, the client's sleep will not be disturbed. Taking furosemide in the morning has no effect on preventing electrolyte imbalances or retarding rapid drug absorption. The client should not accumulate excessive fluids throughout the night

The nurse is administering an IV potassium chloride supplement to a client who has heart failure. When developing a plan of care for this client, the nurse should consider that: • hyperkalemia will intensify the action of the client's digoxin preparation. • the administration of the IV potassium chloride should not exceed 10 mEq/h or a concentration of 40 mEq/L. • metabolic alkalosis will increase the client's serum potassium levels. • the client's potassium levels will be unaffected by a potassium-sparing diuretic.

the administration of the IV potassium chloride should not exceed 10 mEq/h or a concentration of 40 mEq/L. When administering IV potassium chloride, the administration should not exceed 10 or a concentration of 40 via a peripheral line. These limits are extremely important to prevent the development of hyperkalemia and the possibility of cardiac dysrhythmias. In some situations, with dangerously low serum potassium levels, the client may need cardiac monitoring and more than 10 mEq (mmol/L) of potassium per hour. Potassium-sparing diuretics may lead to hyperkalemia because they affect the kidney's ability to excrete excess potassium. Metabolic alkalosis can cause potassium to shift into the cells, thus decreasing the client's serum potassium levels. Hypokalemia can lead to digoxin toxicity.

The nurse interviews a 22-year-old female client who is scheduled for abdominal surgery the following week. The client is obese and uses estrogen-based oral contraceptives. This client is at high risk for development of: • thrombophlebitis. • atherosclerosis. • Raynaud's disease. • diabetes.

thrombophlebitis. The data suggest an increased risk of thrombophlebitis. The risk factors in this situation include abdominal surgery, obesity, and use of estrogen-based oral contraceptives. Risk factors for atherosclerosis include genetics, older age, and a high-cholesterol diet. Risk factors for diabetes include genetics and obesity. Risk factors for vasospastic disorders include cold climate, age (16 to 40), and immunologic disorders.

The nurse is assessing a client who is at risk for cardiac tamponade from chest trauma sustained in a motorcycle accident. What is the client's pulse pressure if his blood pressure is 108/82 mm Hg? Record your answer using a whole number.

• 26 Pulse pressure is the difference between systolic and diastolic pressures. Normally, systolic pressure exceeds diastolic pressure by approximately 40 mm Hg. Narrowed pulse pressure, a difference of less than 30 mm Hg, is a sign of cardiac tamponade.

What is the primary goal for the care of a client who is in shock? • Achieve adequate tissue perfusion. • Prevent hypostatic pneumonia. • Preserve renal function. • Maintain adequate vascular tone. A primary outcome for the care of the client in shock is to achieve adequate tissue perfusion, thus avoiding multiple organ dysfunction. The lungs are susceptible to injury, especially acute respiratory distress syndrome. Vasoconstriction occurs as a compensatory mechanism until the client enters the irreversible stage of shock.

• Achieve adequate tissue perfusion. A primary outcome for the care of the client in shock is to achieve adequate tissue perfusion, thus avoiding multiple organ dysfunction. The lungs are susceptible to injury, especially acute respiratory distress syndrome. Vasoconstriction occurs as a compensatory mechanism until the client enters the irreversible stage of shock.

A client is admitted with a 6.5-cm thoracic aneurysm. The nurse records findings from the initial assessment in the client's chart, as shown. At 1030, the client has sharp midchest pain after having a bowel movement. What should the nurse do first? BP: 160/90 HR: 74 BPM Resp: 20 • Assess the client's vital signs. • Administer pain medication as prescribed. • Assess the client's neurologic status. • Contact the health care provider (HCP).

• Assess the client's vital signs. The size of the thoracic aneurysm is rather large, so the nurse should anticipate rupture. A sudden incidence of pain may indicate leakage or rupture. The blood pressure and heart rate will provide useful information in assessing for hypovolemic shock. The nurse needs more data before initiating other interventions. After assessment of vital signs, neurologic status, and pain, the nurse can then contact the HCP.

A client diagnosed with decompensated heart failure has the nursing diagnosis of risk for anxiety related to fear of death or debilitation. What nursing interventions will help alleviate anxiety? *Select all that apply.* • Encourage the client to ask questions. • Explain in advance all procedures and routine regimens. • Provide a calm environment. • Reposition the client every two hours. • Provide a therapeutic mattress or bed.

• Encourage the client to ask questions. • Explain in advance all procedures and routine regimens. • Provide a calm environment. • Encouraging the client to ask questions provides an open forum for discussion with the client, helping to promote trust and alleviate anxiety. • By providing information in advance about what interventions are being performed and why, the client will have reduced anxiety about the care provided. This will also increase her knowledge of her condition, making her feel more secure. • A calm environment decreases additional anxiety. • Changing position frequently prevents formation of pressure ulcers by decreasing the amount of time there is pressure on any area. This promotes tissue integrity. • Pressure-redistribution mattresses and beds are available to decrease the pressure on the sacrum when the client is sitting up in bed. This promotes tissue integrity.

- Just beneath the right atrium - Consists of transitional cells (T-cells) - Controlled by the sympathetic and parasympathetic nervous system

Atrioventricular (AV) junction

A client has a blockage in the proximal portion of a coronary artery. After learning about treatment options, the client decides to undergo percutaneous transluminal coronary angioplasty (PTCA). During this procedure, the nurse expects to administer an: antibiotic. anticoagulant. antihypertensive. anticonvulsant.

Correct response: anticoagulant. Explanation: During PTCA, the client receives heparin, an anticoagulant, as well as calcium agonists, nitrates, or both, to reduce coronary artery spasm. Nurses don't routinely give antibiotics during this procedure; however, because the procedure is invasive, the client may receive prophylactic antibiotics to reduce the risk of infection. An antihypertensive may cause hypotension, which should be avoided during the procedure. An anticonvulsant isn't indicated because this procedure doesn't increase the risk of seizures.

• Alcohol abuse • Chemo • Infection • Inflammation • Poor nutrition • ↓ CO causes symptoms • DOE (dypnea on exertion) • Fatigue • Palpitations

Cardiomyopathy Causes

• Synchronized counter shock • Only for VF & pulseless VT in emergencies • Afib may be cardioverted --Anticoags for 4-6 wks --TEE to ensure no clots --One electrode on the left and another on the right --Synchronize the HR --Shock after the T wave ----Assess VS, look for burns, monitor rhythm, oxygen

Cardioversion


संबंधित स्टडी सेट्स

Computer Organization and Architecture: Chapter 3 - A Top-Level View of Computer Function and Interconnection

View Set

management information systems test 1

View Set

CHAPTER 17—WORKING CAPITAL MANAGEMENT

View Set

Real Estate Wrong Questions from Practice Tests

View Set